NBME 28 Q Solved

You might also like

Download as pdf or txt
Download as pdf or txt
You are on page 1of 201

Tutorial Section: Screen 10 of 10 National Board of Medical Examiners® Time Remaining:

Tutorial 14 min 18 sec

This concludes the tutorial.

Click " Next" to begin your assessment.

7
L J

Previous Next Help Pause


Exam Section 1: Item 1 of 50 National Board of Medical Examiners^ Time Remaining:
9 Mark Comprehensive Basic Science Self-Assessment 1 hr 14 min 58 sec

1 . A 25 - year- old man is brought to the emergency department because of severe abdominal pain , nausea, and vomiting for 1 hour . The pain originates in the left flank and radiates to his groin. His putse is 100/min, respirations are 18/min, and blood pressure is
150/100 mm Hg. Physical examination shows tenderness of the left flank and the left lower quadrant of the abdomen . Bowel sounds are mildly hypoactive . Test of the stoof for occult blood is negative. Which of the following best explains these findings ?

. A ) Colon neoplasm
B ) Diverticulitis
O C ) Epididymitis
0 D ) Renalinfarction
. E ) Torsion of the testis
F ) Ureteral calculus

Next Lab Values Calculator


9
Review
0
Help
0
Pause
Exam Section 1: Item 2 of 50 National Board of Medical Examiners^ Time Remaining:
9 Mark Comprehensive Basic Science Self-Assessment 1 hr 13 min 57 sec

2 . Which of the following types of sensory information is compromised by lesions of the structure at site X in the photograph shown?

A ) Conscious proprioception
B ) Pain sensation
0 C ) Two- point discrimination
D ) Unconscious proprioception
E:) Vibration sense

& * f*
Previous Next Lab Values Calculator Review Help Pause
Exam Section 1: Item 3 of 50 National Board of Medical Examiners^ Time Remaining:
9 Mark Comprehensive Basic Science Self-Assessment 1 hr 13 min 51 sec

3 . Moving the forearm against resistance from palm-down to palm-up (supination) position requires the use of which of the following muscles ?

3 A ) Biceps brachii
B ) Brachialis
O C ) Triceps
D ) Flexor carpi radialis
! J E ) Pronator teres

Previous Next Lab Values Calculator


9
Review
0
Help
0
Pause
Exam Section 1: Item 4 of 50 National Board of Medical Examiners^ Time Remaining:
® Mark Comprehensive Basic Science Self-Assessment 1 hr 13 min 42 sec

4 . A 35 - year- old man is admitted to the hospital because of a 5 -day history of fever and dyspnea He underwent a bone marrow transplantation G months ago : the procedure was complicated by severe graft-versus-host disease. His temperature is 38 - C
(100.4T ) . and respirations are 30/min . Scattered crackles are heard on auscultation of the chest . A chest x-ray shows patchy infiltrates. A transbronchial biopsy specimen shows findings consistent with cytomegalovirus infection . Intravenous administration of
ganciclovir is begun. This drug interferes with the function of which of the following enzymes ?

A ) DNApolymerase
3 B ) Integrase
o C ) Reverse transcriptase
3 0 ) RNA polymerase
J E } Thymidine kinase

Previous *
Next Lab Values Calculator
m
Review
O
Help
m
Pause
Exam Section 1: Item 5 of 50 National Board of Medical Examiners^ Time Remaining:
9 Mark Comprehensive Basic Science Self-Assessment 1 hr 13 min 36 sec

o
T> 200-
ȣ
£ S 100-
<D
co
0
25 27 1
Cycle (day)
M - menstruation

5 . The graph shows changes in serum estradiol concentration during a normal menstrual cycle . Which of the following ovarian cells is primarily responsible for the aromatizaiion of androstenedione to estradiol at the time indicated by the arrow ?

A ) Granulosa
'

B ) Luteaf
. ) G ) Stromal
D ) Theca externa
E ) Theca interna

O
Previous * Next Lab Values
r-
Calculator
o
Review
o
Help
o
Pause
Exam Section 1: Item 6 of 50 National Board of Medical Examiners^ Time Remaining:
® Mark Comprehensive Basic Science Self-Assessment 1 hr 13 min 32 sec

6 . A 14 - year- old girl is brought to the physician by her parents because of a 1-month history of a rash that appears with sun exposure Her parents tell the physician that she has been eating fiittle food . Physical examination shows a pruritic rash on the exposed
,

areas of the body. Her serum tryptophan concentration is decreased Urine studies show increased excretion of amino acids : predominantly alanine , isoleucine , leucine, phenylalanine: tryptophan, and valine. Production of which of the following vitamins is
most likely impaired in this patient?

A ) Niacin
0 B ) Vitamin B , (thiamine)
0 G ) Vitamin B (riboflavin)
2

0 0 ) Vitamin B (pantothenic add )


5

E ) Vitamin C

Previous *
Next Lab Values Calculator
m
Review
O
Help
m
Pause
Exam Section 1: Item 7 of 50 National Board of Medical Examiners^ Time Remaining:
® Mark Comprehensive Basic Science Self-Assessment 1 hr 13 min 29 sec

7 . Aminoglycoside antibiotics are used for their synergistic action against bacteria in combination with other agents . These antibiotics demonstrate in vitro synergy for several bacterial species when combined with which of the following classes of antibiotics ?
,

3 A ) Fluoroquinolones
S ) Macrolides
O C ) Penicillins
D ) Rifamycrns
! J E ) Tetracyclines

Previous *
Next Lab Values Calculator
m
Review
O
Help
m
Pause
Exam Section 1: Item 8 of 50 National Board of Medical Examiners^ Time Remaining:
® Mark Comprehensive Basic Science Self-Assessment 1 hr 13 min 26 sec

8 . A 16 - year- old boy with moderate intellectual disability is brought to the physician for a routine examination. There is a family history of mild and moderate intellectual disability in his mother and brother, respectively. Physical examination shows a long face ;

prominent ears, and moderately enlarged testicles. Which of the following best describes the genetic mechanism of this patient' s disorder ?

. A ) Mutation in a mitochondrial gene


8 ) Presence of an extra sex chromosome
. J C ) Translocation of a portion of an autosome
3 D ) Trinucleotide repeat mutation on the X chromosome
'

. E } Trisomy of an autosome

O 9 0 0
Previous *
Next Lab Values Calculator Review Help Pause
Exam Section 1: Item 8 of 50 National Board of Medical Examiners^
® Mark Comprehensive Basic Science Self-Assessment Mease Wait
9 . A 75 - year- old woman comes to the physician because of a 3-month history of an enlarging lesion on her face . Physical examination shows a 1 5 -cm , brown-black , mottled, scaly lesion with irregular borders. Microscopic examination of a biopsy specimen of the
,

lesion shows atypical melanocytes spread along the basilar layer of the epidermis . Which of the following is the most likely cause of these findings ?

A ) Acanthosis nigricans
8 ) Actinic keratosis
C ) Compound nevus
D ) Lentigo maligna
E ) Seborrheic keratosis

O
Previous *
Next Lab Values Calculator
m
Review
O
Help
m
Pause
Exam Section 1: Item 10 ol 50 National Board of Medical Examiners^ Time Remaining:
9 Mark Comprehensive Basic Science Self-Assessment 1 hr 13 min 15 sec

10 . A physician wishes to determine the proportion of newborns delivered at a local hospital who had a diagnosis of congenital heart disease within the past year . Which of the following statistical measurements best describes these data ?

Q A ) Attributable risk
S ) Incidence
O C ) Odds ratio
D ) Prevalence
' J E ) Relative risk

Previous *
Next Lab Values Calculator
m
Review
O
Help
m
Pause
Exam Section 1: Item 11 of 50 National Board of Medical Examiners^ Time Remaining:
® Mark Comprehensive Basic Science Self-Assessment 1 hr 13 min 11 sec

11 . A 25 -year- old woman comes to the physician after her blood pressure was found to be 130/105 mm Hg at a health fair . She takes no medications . There is no family history of hypertension. Her last menstrual period was 1 week ago . Her blood pressure today
is 180/ 110 mm Hg . Bilateral abdominal bruits are heard . Treatment with an angiotensin-converting enzyme ( ACE) inhibitor will most likely have which of the following acute effects on this patient' s renal function?

. A ) Decreased concentrating ability secondary to renal angioedema


B ) Decreased glomerular filtration rate secondary to dilation of efferent arterioles
. 5 C ) Decreased renal blood flow secondary to dilation of afferent arterioles
!
'

D ) increased concentrating ability secondary to a change in permeability of the collecting duct


. E ) Interstitial nephritis secondary to allergic drug reaction

Previous *
Next Lab Values Calculator
m
Review
O
Help
m
Pause
Exam Section 1: Item 12 ol 50 National Board of Medical Examiners^ Time Remaining:
9 Mark Comprehensive Basic Science Self-Assessment 1 hr 12 min 57 sec

12 A 16 -year- ofd boy is brought to the physician because of a 3- month history of shortness of breath while playing sports. He has no shortness of breath at rest . He says " Whenever I run around I cough, so i don't want to be on the basketball team anymore. " He
7

takes no medications and has no known allergies . There is a family history of hypertension and asthma . He is 165 cm (5 ft 5 in) tall and weighs 68 kg (150 lb ) : BMI is 25 kg /m 2. His respirations are 12/min, and blood pressure is 115 /75 mm Hg . Cardiac
examination shows no abnormalities except for a midsystolic click at the apex. The lungs are clear to auscultation of the chest . Which of the following best explains this patient' s symptoms ?

A ) Deconditioning
3 B ) Exercise-induced asthma
o G ) Malingering
3 0 ) Mitral valve prolapse
J E } Thyroid disease

O
Previous Next Lab Values Calculator
m
Review
O
Help
m
Pause
Exam Section 1: Item 13 of 50 National Board of Medical Examiners^ Time Remaining:
9 Mark Comprehensive Basic Science Self-Assessment 1 hr 12 min 53 sec

13. A 25 -year- ofd man comes to the physician because of a 3- day history of pain and swelling of his right leg . He has no history of major medical illness or recent trauma . Examination of the right lower extremity shows edema and tenderness . Duplex
ultrasonography of the right lower extremity shows a thrombus extending into the superficial femoral vein . Further studies show protein C deficiency. Inactivation of which of the foliowing coagulation factors is most likely as a result of this deficiency in this
patient?

A ) Factors V (proaccelebn) and VIII ( antihemophilic factor)


3 B ) Factors V (proaccelebn ) and IX ( plasma thromboplastin component)
O C ) Factors V (proaccelebn) and XI (plasma thromboplastin antecedent)
3 D ) Factors VIII (antihemophilic factor ) and IX (plasma thromboplastin component)
Q E ) Factors VIII (antihemophilic factor) and XI (plasma thromboplastin antecedent)
( J F ) Factors IX (plasma thromboplastin component ) and XI (plasma thromboplastin antecedent)

Previous *
Next Lab Values Calculator
m
Review
O
Help
m
Pause
Exam Section 1: Item 14 ol 50 National Board of Medical Examiners^ Time Remaining:
® Mark Comprehensive Basic Science Self-Assessment 1 hr 12 min 50 sec

14 . Which of the following is most directly responsible for concentrating testosterone in the lumen of the seminiferous tubules ?

3 A ) Androgen - binding protein

O ^ S ) Follicle-stimulating hormone (FSH)


G ) FSH/gonadotropin-releasing hormone
o Q) Irthibm
' J E ) Luteinizing hormone

O
Previous Next Lab Values Calculator
m
Review
O
Help
m
Pause
Exam Section 1: Item 15 ol 50 National Board of Medical Examiners^ Time Remaining:
0 Mar
*
Comprehensive Basic Science Self-Assessment 1 hr 12 min 47 sec

15. A 23 -year- old woman has had the lesions shown In her mouth for 3 days. She has had frequent similar episodes over the past 15 years . The lesions are exacerbated by spicy, salty: and acidic food and drinks . They last approximately 1 week and resolve
spontaneously. Visits to the dentist seem to trigger the development of the sores. Which of the following is the most likely diagnosis?

A ) Aphthous ulcers
3 B ) Candidiasis
0 C ) Geographic tongue
O 0 ) Koplik spots

^F E ) Leukoplakia
) Lichen planus
'
G ) Psoriasis

o
Previous Next Lab Values Calculator
9
Review
0
Help
0
Pause
Exam Section 1: Item 16 ol 50 National Board of Medical Examiners^ Time Remaining:
® Mark Comprehensive Basic Science Self-Assessment 1 hr 12 min 41 sec

16. A 20-year- old woman comes to the emergency department 30 minutes after slippmg on ice and extending her hand to break her fall. Palpation of the anatomic snuff -box produces pain . A wrist x - ray is most likely to show a fracture of which of the following
carpal bones?

. A ) Scaphoid
8 ) Lunate
. ) C ) Triquetrum
D ) Pisiform
. . E ) Trapezium
F ) Trapezoid
. ) G ) Capitate
H ) Hamate

O
Previous *
Next Lab Values Calculator
m
Review
O
Help
m
Pause
Exam Section 1: Item 17 of 50 National Board of Medical Examiners^ Time Remaining:
® Mark Comprehensive Basic Science Self-Assessment 1 hr 12 min 38 sec

17. A 7 G -year- ofd man undergoes laparotomy for resection of an abdominal aortic aneurysm . During the procedure, an incidental finding of acquired colonic diverticula is made. The diverticula in this patient are most likely present in which of the following?

A ) Ascending coion
i S ) Cecum

Q G ) Descending colon
D ) Sigmoid colon
! J E ) Transverse colon

Previous *
Next Lab Values Calculator
m
Review
O
Help
m
Pause
Exam Section 1: Item 13 ol 50 National Board of Medical Examiners^ Time Remaining:
9 Mark Comprehensive Basic Science Self-Assessment 1 hr 12 min 19 sec

18. During a period of 36 hours an 80 -year - old woman has increasingly severe abdominal pain followed by fever chills , tachycardia: hypotension and . finally shock . Blood cultures grow Escherichia coti. Her condition worsens and . despite supportive therapy
, ,

and antibiotics she dies 4 days after the onset of the illness. Which of the following is the most likely cause of the initial hypotension?
;

. A ) Excessive production of nitric oxide


B ) Generation of hydrogen peroxide
o C ) Hemorrhage
! D ) Induction of endothelial adhesion molecules
. E } Platelet aggregation

Previous *
Next Lab Values Calculator
m
Review
O
Help
m
Pause
Exam Section 1: Item 19 of 50 National Board of Medical Examiners^ Time Remaining:
® Mark Comprehensive Basic Science Self-Assessment 1 hr 12 min 16 sec

19. A 65 -year- old woman comes to the physician because of a 3-month history of headache, weakness of her arms , and left flank pain : she also has had a 14 -kg ( 31-Eb ) weight loss during this period. Physical examination shows weakness of the proximal upper
and lower extremity muscles. There is augmentation of strength with repetitive testing of the deltoid muscles. An MRI of the brain shows a single well-demarcated mass surrounded by edema in the right frontal lobe. A stereotactic biopsy specimen of the lesion
shows a malignant; smalt blue cell neoplasm that expresses cytokeratin , chromogranin, and synaptophysin . Which of the following is the most likely diagnosis?

A ) Anaplastic ependymoma
3 B ) ExtranodaE primary central nervous system lymphoma
o C) Glioblastoma multiforme
3 0 ) Primary cerebral neuroblastoma
E } Pulmonary small cell carcinoma metastatic to the brain

®
Previous
0
Next Lab Values Calculator
0
Review
0
Help
0
Pause
Exam Section 1: Item 20 ol 50 National Board of Medical Examiners^ Time Remaining:
® Mark Comprehensive Basic Science Self-Assessment 1 hr 12 min 12 sec

20 . Failure of normal differentiation of the endoderm in the embryonic lung bud is most likely to affect the development of which of the following?

3 A ) Capillary patterns
3 S ) Cartilage in bronchi
O Gj Smooth muscle on the bronchi
J D ) Surfa etant sec retion
3 E ) Tracheal rings

Previous Next Lab Values Calculator


o
Review
o
Help
o
Pause
Exam Section 1: Item 21 ol 50 National Board of Medical Examiners^ Time Remaining :
® Mark Comprehensive Basic Science Self-Assessment 1 hr 12 min 9 sec

21 . A 67-year- old woman comes to the physician for a health maintenance examination . Her brother and mother have a history of colon cancer . The physician recommends colonoscopy: but the patient says that she would prefer only for her stool to be tested for
biood . The physician explains that testing the stool for occult blood is not appropriate in this case. The physician is most likely concerned about which of the following regarding this test?

. A ) Low sensitivity
8 ) Low specificity
. J C ) Potential for a false -positive result
3 D ) Uncertain negative predictive value
. E ) Uncertain positive predictive value

Previous *
Next Lab Values Calculator
m
Review
O
Help
m
Pause
Exam Section 1: Item 22 ol 50 National Board of Medical Examiners^ Time Remaining :
® Mark Comprehensive Basic Science Self-Assessment 1 hr 12 min 5 sec

22 A 5 - year- old boy is brought to the emergency department after ingesting 10 oz of a household cleaning solvent He is treated for acute hepatic and renal failure for 1 week and then discharged . During the next montfr regeneration of this boy's mature
hepatocytes and renal tubular epithelial cells will be accomplished mostly by which of the following mechanisms ?

. A ) Activation of stem cells to enter G 1 phase of the cell cycle


8 ) Decreased apoptosis at GrM transition of the cell cycle
. J G ) Recruitment of cells from G 0 into the cell cycle
3 D ) Shortened time for progression of cells through the cell cycle

. E ) Terminal differentiation by celts exiting from the cell cycle

O 0 I* 0 0 0
Previous Next Lab Values Calculator Review Help Pause
Exam Section 1: Item 23 of 50 National Board of Medical Examiners^ Time Remaining :
® Mark Comprehensive Basic Science Self-Assessment 1 hr 12 min 4 sec

23. A 17-year- old boy is brought to the physician by his mother because she is concerned that his puberty is delayed . The mother states . "He is so short. His father is 6 feet 5 inches talk I don ' t understand why he has not had his growth spurt ." When the mother
leaves the room, the patient states : ' I ' m fine . I don't know what ' s the matter with her. She wants me to be tall like my dad ." The patient is 175 cm (5 ft 9 in ) tali and weighs 70 kg (155 lb): BMI is 23 kg / m 2 Sexual development is Tanner stage 4 . in addition to
reassuring the mother that her son is fine, which of the following is the most appropriate initial statement by the physician to the mother?

A ) "Since your son is fine with his height you should try to accept him as he is. "
3 B ) "Tell me more about your concerns about your son ' s height ."
o C ) "Well do some blood tests just to be sure that all your son's hormone levels are okay " .

3 O ) "Your son is average for his height and weight . "


J E ) "Your son is not going to be any taller."

Previous *
Next Lab Values Calculator
m
Review
O
Help
m
Pause
Exam Section 1: Item 24 ol 50 National Board of Medical Examiners^ Time Remaining :
® Mark Comprehensive Basic Science Self-Assessment 1 hr 12 min 2 sec

24. A 34 -year- old woman is admitted to the hospital for treatment of pulmonary tuberculosis . Infliximab therapy was initiated 6 months ago for severe Crohn disease. This pharmacotherapy most likely inhibited which of the following immunologic functions in this
patient?

. A ) Activation of nuclear factor K8 to induce expression of interleukin -10 ( IL -10)


8 ) Direct toxicity to the causal organism
. J C ) Maintenance of granulomas
3 D ) Recruitment of segmented neutrophils to ingest and kill the bacteria
'

(_ y E ) Stimulation of B lymphocytes to produce neutralizing antibodies against the causal organism

Previous *
Next Lab Values Calculator
m
Review
O
Help
m
Pause
Exam Section 1: Item 25 ol 50 National Board of Medical Examiners^ Time Remaining:
® Mark Comprehensive Basic Science Self-Assessment 1 hr 11 min 53 sec

25. An 8 -year - old boy continues to bleed excessively after tooth extraction . Prothrombin time , bleeding time and platelet count are within the reference range . Partial thromboplastin time is prolonged but corrects after addition to the assay chamber of plasma
,

from a patient with hemophilia A. Which of the following is the most likely diagnosis?

. A ) Acute disseminated intravascular coagulation


8 ) Pactor V (proaccelerin ) deficiency
. J C ) Factor VII ( proconvertin ) deficiency
D ) Hemophilia A
. . E ) Hemophilia B
F ) Immune thrombocytopenic purpura
. J. G ) von Willebrand disease

O
Previous *
Next Lab Values Calculator
m
Review
O
Help
m
Pause
Exam Section 1: Item 26 ol 50 National Board of Medical Examiners^ Time Remaining:
® Mark Comprehensive Basic Science Self-Assessment 1 hr 11 min 56 sec

26. The diagram shows the major factors that determine blood pressure . Which of the following labeled factors is affected most by an aradrenergic antagonist?

Blood pressure
^ Central nervous system


Cardiac output x
Peripheral resistance
farter id fat)

©
Stroke volume
^Heort rale
J

j
^
" Contractility Venous return

Blood volume Capacitance vessel tone


(v &nutar )

©
O A)
O 8)
O C)
O D)
OE)

Previous Next Lab Values Calculator


m
Review
O
Help
m
Pause
Exam Section 1: Item 27 of 50 National Board of Medical Examiners^ Time Remaining:
® Mark Comprehensive Basic Science Self-Assessment 1 hr 11 min 54 sec

r
f
A . Jl /1
/ \

<
.

5 iVtt VI ; ^
|f

v 05
i
II uVL V2 VJ

R r
*

m nVF V? v
*


4

_
II
V~- I --
T

— I V ^ -
H

vs ^ 4ir4 . J
^
A

27 . A previously healthy 21- year -old woman comes to the office because of a 2-month history of shortness of breath and fatigue. Her most recent menstrual period was 3 months ago. Menses previously had occurred at regular 28 -day intervals . She tells the
,

physician that she thinks she may be pregnant . She takes no medications and has not seen a physician for several years . She appears healthy. She is 160 cm (5 ft 3 in) tall and weighs 54 kg (120 ib ) ; BMI is 21 kg /m 2 Vital signs are within normal limits. The
lungs are dear. Cardiac examination shows a normal S ( a widely split S 2 that does not change with respiration , and a grade 3/6 holosystoMc murmur that is loudest at the lower left sternal border and radiates to the upper left sternal border. EGG is shown.
f

The most likely cause of these ffndings is dysfunction of which of the following structures?

A ) Atrial septum
3 B ) Ductus arteriosus
Q C ) Interventricular septum
D ) Pulmonic valve
E ) Tricuspid valve

Previous *
Next Lab Values Calculator
m
Review
O
Help
m
Pause
Exam Section 1: Item 28 ol 50 National Board of Medical Examiners^ Time Remaining:
® Mark Comprehensive Basic Science Self-Assessment 1 hr 11 min 52 sec

‘- -i - H
v:
-v
- *.’ vy -
ft

/ ^ -p
J
J
VJ
k
%

8 ..
-- .
b
r
: mrnm *
fi* * " -
T 7
I aw
.
£ .J
i i

a B
V
. % - -kr
rt

v- . r -
w

-> WWHB
i

*. it
_

: r’v
»- :
-' ^
dS
V
?^ ,.v
J

_
^ .V ’ d> S
^
&
IS
n
11

"H H - J

*
T
* *
IT “ S
I VP
"
i

-.
J
1 P
s i" 3

r 1
ii
p
*

K i
i
i a
l.
i s
f t
•J~ r
J r*
9" - J
bfa
'

In
P
J
b

X
n

P
9
« ]31

28. A 53 -year- old man has had progressive difficulty swallowing for the past 3 months. He has a 10 -year history of heartburn with esophageal regurgitation of gastric contents. Tissue obtained on biopsy of the lower third of the esophagus is shown . Which of the
following best describes the nature of this lesion?

A } Basal zone hyperplasia of submucosal glands


! J B ) Intestinal metaplasia of squamous epithelium
O G ) Malignant transformation of epithelium into squamous carcinoma
' 0 ) Squamous metaplasia of submucosal glands

«1
Previous
0
Next
(IS
Lab Values Calculator
r
Review
r
Help
r»;
Pause
Exam Section 1: Item 29 of 50 National Board of Medical Examiners^ Time Remaining:
® Mark Comprehensive Basic Science Self-Assessment 1 hr 11 min 45 sec

29. A 50-year- old man comes to the emergency department because of a 2 -week history of progressive shortness of breath. His pufse is 90/min , respirations are 26/ min: and blood pressure is 120/S 0 mm Hg . Physical examination shows no other abnormalities .
Laboratory studies show :
Arterial Pco 2 30 mm Hg
Arterial Po 2 96 mm Hg
Arterial 02 content 12 vol% (N=17%-21%)
Mixed venous Po 2 36 mm Hg
Mixed venous 02 content 8 vol% (N=10%-1:6% )

Which of the following is the most likely explanation for these findings ?

A ) Anemia
B ) Drug-induced alveolar hypoventilation
C ) Residence at a high altitude
D ) Severe regional mismatching of alveolar ventilation and pulmonary capillary perfusion
E ) Voluntary hyperventilation

Previous *
Next Lab Values Calculator
m
Review
O
Help
m
Pause
Exam Section 1: Item 30 ol 50 National Board of Medical Examiners^ Time Remaining:
® Mark Comprehensive Basic Science Self-Assessment 1 hr 11 min 46 sec

30 . An investigator is studying the effects of triiodothyronine (T ) and thyroxine (TJ in hepatocytes in an experimental animal model . Which of the following best describes the action of these thyroid hormones on this target tissue ?
^
3 A ) Both T 3 and T 4 bind to the melanocortin 2 receptor on the cell surface
J B ) Both T 3 and T 4 enter the nucleus
( G ) T 3 is converted to T 4 in the cytosol
J D ) Thyroid hormone receptors preferentially bind T 4 over T 3

O
Previous Next Lab Values Calculator
m
Review
O
Help
m
Pause
Exam Section 1: Item 31 ol 50 National Board of Medical Examiners^ Time Remaining:
® Mark Comprehensive Basic Science Self-Assessment 1 hr 11 min 42 sec

31. A female newborn delivered at 38 weeks' gestation to a 28 - year- old woman gravida 1 , para 1 develops respiratory distress . Pregnancy and delivery were uncomplicated amniotic fluid was clear and the placenta was normal. Fetal ultrasonography and MRf
;
'
,

at 34 weeks’ gestation showed a congenital diaphragmatic hernia with evidence of small bowel and stomach herniation into left hemithorax . She is 52 cm (20.4 in ) long and weighs 3500 g ( 7 lb 11 oz). Her temperature is 37.5°C (99.ST ), pulse is 138/ min .
:

respirations are 50/min, and blood pressure is 70/55 mm Hg . Physical examination shows peripheral cyanosis that improves after administration of oxygen via endotracheal intubation . Breath sounds are decreased on the left . Cardiac examination shows
normal heart sounds without murmurs . Which of the following complications of the described pathology is most likely to be life threatening to this newborn?

A ) Active pneumonia
. B ) Alveolar edema
0 C ) Amniotic embolism
0 0 ) Inadequate surfactant synthesis
E ) Pulmonary hypoplasia

Previous *
Next Lab Values Calculator
m
Review
O
Help
m
Pause
Exam Section 1: Item 32 ol 50 National Board of Medical Examiners^ Time Remaining:
® Mark Comprehensive Basic Science Self-Assessment 1 hr 11 min 35 sec

32 An 18-hour -old male newborn Is 61 cm (24 in) long and weighs 5443 g ( 12 lb). His mother has type 1 diabetes mellitus . His serum glucose concentration is 20 rrig/ dL Which of the following fetal conditions immediately prior to birth most likely precipitated the
newborn's postnatal hypoglycemia ?

. A ) Decreased gluconeogenesis
8 ) Decreased glycogen concentration
. J C ) Decreased glycogen synthetase activity
3 D ) Decreased serum insuEin concentration
. E } Increased serum insulin-like growth factor

O
Previous *
Next Lab Values Calculator
m
Review
O
Help
m
Pause
Exam Section 1: Item 33 of 50 National Board of Medical Examiners^ Time Remaining:
® Mark Comprehensive Basic Science Self-Assessment 1 hr 11 min 31 sec

33. The synthesis of the enzymes necessary for the replication of the genome occurs during which of the following phases of the cell cycle ?

O A)
O B)
O C)
O D)
O E)

Previous *
Next Lab Values Calculator
m
Review
O
Help
m
Pause
Exam Section 1: Item 34 ol 50 National Board of Medical Examiners^ Time Remaining:
® Mark Comprehensive Basic Science Self-Assessment 1 hr 11 min 29 sec

34. A 22 -year- old woman is brought to the emergency department in a semicomatose condition after collapsing near the end of running a marathon . Her prerace weight was 47 kg (103 lb). She now weighs 50 kg ( 110 lb). Her pulse is 115/min, respirations are
15/min, and blood pressure is 90/50 mm Hg . Physical examination shows cool , dry skin . She is responsive to painful stimuli. Laboratory studies show:
Serum
Na + 116 mEq / L
K -
ci -
4.8 mEq/ L
89 mEq/L
HCO 3- 22 mEq/L
Urea nitrogen 22 mg/dL
Glucose 101 mg/dL
Creatinine 1 mg/dL

This patient' s condition is most likely due to which of the following?

o A ) Decreased ADH (vasopressin)


6 } Decreased aldosterone
Q C ) Excessive fluid intake
! _ D ) Inadequate fluid intake
O E ) Increased aldosterone

Previous *
Next Lab Values Calculator
m
Review
O
Help
m
Pause
Exam Section 1: Item 35 ol 50 National Board of Medical Examiners^ Time Remaining:
® Mark Comprehensive Basic Science Self-Assessment 1 hr 11 min 26 sec

35. A 36-year- old woman with type 2 diabetes mellitus comes to the physician for a follow -up examination. Current medications include a sulfonylurea . She is 173 cm ( 5 ft 3 in) tall and weighs 95 kg (210 lb ) : BMI is 32 kg /m 2 Physical examination shows
acanthosis nigricans . Treatment with metformin is most likely to produce which of the following effects in this patient?

. A ) Decrease intestinal carbohydrate digestion


B ) increase beta -cell insulin secretion
. J C ) Increase deposition of adipocyte fat
3 D ) increase hepatic triglyceride synthesis
'

. E } Inhibit hepatic gluconeogenesis

O
Previous *
Next Lab Values Calculator
m
Review
O
Help
m
Pause
Exam Section 1: Item 36 ol 50 National Board of Medical Examiners^ Time Remaining:
® Mark Comprehensive Basic Science Self-Assessment 1 hr 11 min 23 sec

36. A 23-year- old man in the emergency department has apnea and pinpoint pupils. Needle tracks are present on his arms. Activation of which of the following opioid receptors in the central nervous system is most likely to be responsible for the apnea?

O A) 5
O B) K
O C) u
O D) a

O
Previous *
Next Lab Values Calculator
m
Review
O
Help
m
Pause
Exam Section 1: Item 37 of 50 National Board of Medical Examiners^ Time Remaining:
® Mark Comprehensive Basic Science Self-Assessment 1 hr 11 min 21 sec

37. A 5 - year- old girl with premature sexual development is diagnosed with precocious puberty. Pelvic examination shows a mass consistent with an ovarian tumor. Laboratory studies show decreased serum concentrations of gonadotropins and a marked
increase in circulating estrogens. The ovarian tumor is most likely derived from which of the following cell types?

. A ) Endothelial cells
8 ) Germinal epithelium
. ) G ) Granulosa cells
0 D ) Stromal fibroblasts
E ) Thecal cells

Previous *
Next Lab Values Calculator
m
Review
O
Help
m
Pause
Exam Section 1: Item 33 ol 50 National Board of Medical Examiners^ Time Remaining:
® Mark Comprehensive Basic Science Self-Assessment 1 hr 11 min 19 sec

38. A healthy 22 -year- old woman undergoes testing to determine whether she is a suitable kidney donor for her 25-year -old brother with end -stage renal disease caused by type 1 diabetes meilitus. Immunologic studies show that she is human leukocyte antigen
(HLAJ-DR3/ DR6 positive and her brother is HLA-DR 3/DR4 positive. To determine the extent of alioreactivity: a mixed lymphocyte reaction is done using irradiated stimulator cells isolated from the donor and responder cells isolated from the recipient . The T
;

lymphocytes that proliferate in these cultures will most likely react with which of the following HLA types?

A ) DR3 only
O B ) OR3 and DR4
o C ) DR3 and DR6
o 0 ) 0R4 only
E ) DR4 and DR6
O F ) 0R6 only

O
Previous *
Next Lab Values
I*
Calculator
m
Review
O
Help
m
Pause
Exam Section 1: Item 39 of 50 National Board of Medical Examiners^ Time Remaining:
9 Mark Comprehensive Basic Science Self-Assessment 1 hr 11 min 17 sec

39. A 67-year- old man with poorty controlled unstable angina is about to undergo coronary angiography with stent placement Prior to the procedure treatment is initiated with aspirin and a drug that inhibits platelet interaction with fibrinogen. This drug is most
,

likely which of the following?

. A ) Abciximab
8 ) Celecoxib
0 C ) Cilostazol
_ D ) Cloptdogrel
. E } Dipyridamole

Previous *
Next Lab Values Calculator
m
Review
O
Help
m
Pause
Exam Section 1: Item 40 ol 50 National Board of Medical Examiners^ Time Remaining:
® Mark Comprehensive Basic Science Self-Assessment 1 hr 11 min 15 sec

40 . Lesch-Nyhan syndrome , an X -linked recessive disease is seen in approximately 1/100.000 males . Which of the following is the expected prevalence of heterozygous females ?
;

O A ) 1/1000
c B ) 1/10,000
o C ) 1/50,000
'

0 ) 1/200 000 ,

o E ) 1/10 000 000


, ,

o
Previous Next Lab Values Calculator
m
Review
O
Help
m
Pause
Exam Section 1: Item 41 ol 50 National Board of Medical Examiners^ Time Remaining:
® Mark Comprehensive Basic Science Self-Assessment 1 hr 11 min 12 sec

41. The specimen shown is from a 65 -year-old man . Which of the following is the most likely diagnosis?

O A ) Acute leukemia
B ) Colonic carcinoma
0 C ) Hepatic cell carcinoma
D ) Leiomyosarcoma
E ) Liposarcoma

Previous Next Lab Values Calculator


o
Review
o
Help
o
Pause
Exam Section 1: Item 42 ol 50 National Board of Medical Examiners^ Time Remaining:
® Mark Comprehensive Basic Science Self-Assessment 1 hr 11 min 10 sec

42 A 45 -year- old man who is able to bicycle 4 5 minutes a day has switched to a rowing machine . After 5 minutes on the machine , he experiences vertigo , lightheadedness . and fatigue of the left upper extremity. Within a few minutes of stopping the rowing
exercise , all symptoms resolve .. Which of the following findings is most likely on physical examination?

O A ) Diastolic murmur at the cardiac apex


8 ) increased jugular venous pressure
. J C ) Pansystolic murmur at the cardiac apex
D ) Right carotid bruit
. E ) Supraclavicular bruit

O
Previous *
Next Lab Values Calculator
m
Review
O
Help
m
Pause
Exam Section 1: Item 43 of 50 National Board of Medical Examiners^ Time Remaining:
9 Mark Comprehensive Basic Science Self-Assessment 1 hr 11 min 8 sec

43. A 55 -year- old man comes to the physician because of a 2 - month history of increasing difficulty swallowing and regurgitation of undigested food . He also has noticed unusual rumbling sounds in his voice that he feels originate in his neck . Physical examination
shows halitosis. A videofluoroscopic swallowing study shows a 4- cm , posterior midline pouch protruding between the thyropharyngeus and cricopharyngeus portions of the inferior pharyngeal constrictor muscle. These muscles are most likely innervated by
which of the following nerves?

A ) Glossopharyngeal nerve
0 B ) Hypoglossal nerve
0 G ) Motor fibers from the vagus nerve
0 0 ) Parasympathetic fibers from the vagus nerve
O & ) Sympathetic fibers from the superior cervical ganglion

Previous *
Next Lab Values Calculator
m
Review
O
Help
m
Pause
Exam Section 1: Item 44 ol 50 National Board of Medical Examiners^ Time Remaining:
9 Mark Comprehensive Basic Science Self-Assessment 1 hr 11 min 6 sec

44. A 5 - year- old girl is brought to the physician because of list; essness fatigue and dulf pain in the right upper quadrant of the abdomen. Her height and weight are below the 25th percentile . Laboratory findings indicate that the content of her p- globin chain is
: ,

15% to 20% of normal . Sequencing of the (3 -globin gene shows a point mutation in a sequence 3 ' to the coding region in which AATAAA is converted to AACAAA . Consequently the amount of mRNA for (3 -globin is decreased to 10% of normal . Which of the
,

following functions in rriRMA synthesis and processing is most fikefy encoded by the sequence AATAAA ?

A ) Capping with GTP


3 B ) Cleavage and pofyadenylation
o C) Silencing of the promoter
3 0 ) Splicing of the initial mRNA transcript in the nucleus
J E } Transport of the mRNA out of the nucleus

Previous *
Next Lab Values Calculator
m
Review
O
Help
m
Pause
Exam Section 1: Item 45 ol 50 National Board of Medical Examiners^ Time Remaining:
9 Mark Comprehensive Basic Science Self-Assessment 1 hr 11 min 3 sec

45. A 7 G -year- old man comes to the physician for a follow-up examination. He has hypertension treated with a p- adrenergic antagonist He lives on a farm in central California and says he has always distilled his own liquor Before retiring f 0 years ago, he
worked in a hat factory and subsequently in a textile factory. He has smoked 2 packs of cigarettes daily for the past 55 years. He tells the physician that he has had several episodes of painful swelling of his right great toe. Physical examination shows several
lesions consistent with gouty tophi over the elbows bilaterally. Laboratory studies show :
Hemoglobin A 1c 5.6%
Serum
Glucose 93 mg/dL
Creatinine 3.2 mg/dL
Uric acid 7.9 mg/dL

The most likely cause of this patient ' s condition is which of the following?

A ) Cigarette smoking

^ B ) Drinking home -distilled liquor


C ) Farming in central California
Q D ) Working in a hat factory
E } Working in a textile factory

o
Previous *
Next Lab Values Calculator
9
Review
0
Help
0
Pause
Exam Section 1: Item 46 ol 50 National Board of Medical Examiners^ Time Remaining:
9 Mark Comprehensive Basic Science Self-Assessment 1 hr 11 min 1 sec

46. A 22 - year- old woman comes to the physician because of a 6-month history of difficulty swallowing . She says that she feels like she is choking on both solids and liquids . She has no pain with
swallowing . She has had a 4.5-kg f 10-lb) weight loss during this time. There is no history of fever or chills. She is not sexualfy active. She does not smoke cigarettes or use illicit drugs. She is 170 cm
(5 1! 7 in) tall and weighs 59 kg ( 130 lb ); EM is 20 kg/m 2 Her vital signs are within normal limits . Physical examination shows no abnormalities . An x-ray of the esophagus is shown . Which of the
following is the most likely explanation for this patient's symptoms ?

O A ) Acid reflux into the lower esophagus


Q B } Atrophy of the smooth muscle in the esophagus
o C ) Inflammatory degeneration of esophageal waif neurons
D ) Longitudinal mucosal tear at the esophagogastric junction
E ) Perforation of the esophageal wall

«r
Previous *
Next Lab Values Calculator
m
Review
O
Help
m
Pause
Exam Section 1: Item 47 ol 50 National Board of Medical Examiners^ Time Remaining:
® Mark Comprehensive Basic Science Self-Assessment 1 hr 10 min 59 sec

47. A 4 S -year- old woman comes to the office because of a 4 -month history of headaches , itchy skim difficulty swallowing, heartburn, chest tightness, pain in her arms and legs and a burning sensation with urination. She has a history of simitar symptoms since
;

the age of 14 years but previous examinations showed no abnormalities . Her vital signs are within normal limits. Physical examination and laboratory studies today show no abnormalities . Which of the following is the most likely diagnosis?

A ) Conversion disorder
8 ) Factitious disorder
. J C ) Illness anxiety disorder (hypochondriasis )
D ) Malingering
. E } Somatic symptom disorder

O
Previous *
Next Lab Values Calculator
m
Review
O
Help
m
Pause
Exam Section 1: Item 43 ol 50 National Board of Medical Examiners^ Time Remaining:
® Mark Comprehensive Basic Science Self-Assessment 1 hr 10 min 56 sec

48. Based on the graph of ju-aminohippurate (PAM) concentration versus PAH secretion, which of the following is lower at point Y than at point X?
Y
80 -
A ) Glomerular filtration rate “O
at
c
B ) PAH clearance £
Q
OJ & X
Q C ) PAH excretion rate n
<
x l
40 —
'

D ) PAH filtered bad <


E ) Renal blood flow
0 T T i
0 20 40 60 80 100
Plasma p -aminohippurate
( PAH ) concentration
mg/dL

Previous Next Lab Values Calculator


o
Review
o
Help
o
Pause
Exam Section 1: Item 49 of 50 National Board of Medical Examiners^ Time Remaining:
® Mark Comprehensive Basic Science Self-Assessment 1 hr 10 min 52 sec

49. A 41-year- old woman is evaluated because of increasingly severe headaches for 6 weeks . Her blood pressure is 160/100 mm Hg while standing and supine . A bruit is heard over the left costovertebral angle. Urinalysis shows no abnormalities. An angiogram
of the left renal artery shows alternating areas of stenosis and aneurysmal dilatation (" string of beads" sign). Which of the following conditions of the renal artery is the most fikely diagnosis?

. A ) Fibromuscular dysplasia
8 ) Hyaline arteriolosclerosis
. J C ) Intimal fibroplasia
3 0 ) Periarterial fibroplasia
. E } Perimedial hyperplasia

O 9 0 0
Previous *
Next Lab Values Calculator Review Help Pause
Exam Section 1: Item 50 ol 50 National Board of Medical Examiners^ Time Remaining:
® Mark Comprehensive Basic Science Self-Assessment 1 hr 10 min 51 sec

50 . A cohort study assessing risk factors for acquisition of infection with a newly identified agent is performed . Only newly diagnosed subjects are eligible , and controls are selected on the basis of age. The results of this study are shown :
Infection Present Infection Absent
Turtle exposure 60 20
No turtle exposure 40 80

Which of the following is the relative risk for the exposure variable ?
,

O A ) 0-7
O B ) 1.0
O C ) 1.7
O P) 2.2
O E ) 3.1

Previous *
Next Lab Values Calculator
m
Review
O
Help
m
Pause
Exam Section 2: Item 1 of 50 National Board of Medical Examiners^ Time Remaining:
9 Mark Comprehensive Basic Science Self-Assessment 1 hr 14 min 55 sec

1 . A 30 - year- old man with HIV infection has been treated with a combination of antiretroviral drugs , including zidovudine (AZT) for 3 years . Laboratory studies show a marked increase in his plasma HIV viral load during the past 3 months. Viral resistance to
;

zidovudine is suspected . A mutation in which of the following is most likely to explain the resistance to zidovudine in this patient ?

A ) Integrase
8 ) Neuraminidase
. C ) Protease
! D ) RNA -dependent DNA polymerase
. E } Thymidine kinase

Next Lab Values Calculator


9
Review
0
Help
0
Pause
Exam Section 2: Item 2 of 50 National Board of Medical Examiners^ Time Remaining:
9 Mark Comprehensive Basic Science Self-Assessment 1 hr 14 min 51 sec

2 . An investigator is studying [ adrenoreceptors in female experimental animals . During the experiment epinephrine is injected intramuscularly into each animal, and the effects on p ^-ad renoreceptors are then observed . Which of the following physiologic effects
^
is most likely to be observed in these animals ?
,

. A ) Increased myocardial contractility


8 ) internal urethral sphincter contraction
O C ) Lipolysis
3 D ) Pilomotor contraction
. . E ) Pupillary dilation
F ) Uterine relaxation

Previous *
Next Lab Values Calculator
m
Review
O
Help
m
Pause
Exam Section 2: Item 3 of 50 National Board of Medical Examiners^ Time Remaining:
9 Mark Comprehensive Basic Science Self-Assessment 1 hr 14 min 43 sec

3 . A 16 - year- old boy is brought to the emergency department because of a 2 -day history of increasingly severe abdominal pain . His temperature is 39 X (182:2X1pulse is 86,'min, respirations are 18/min, and blood pressure is 120/60 mm Hg. Abdominal
examination shows exquisite tenderness of the right lower quadrant. His leukocyte count is 16: 000/mma. An appendectomy is done; the appendix is swollen with a tan exudate on the serosal surface . Which of the fofiowing best characterizes the leukocytosis in
this patient ?

A ) Basophilia
0 B ) Eosinophiiia
0 G ) Lymphocytosis
0 0 ) Monocytosis
J E } Neutrophilia

Previous *
Next Lab Values Calculator
m
Review
O
Help
m
Pause
Exam Section 2: Item 4 of 50 National Board of Medical Examiners^ Time Remaining:
® Mark Comprehensive Basic Science Self-Assessment 1 hr 14 min 46 sec

4 . A 32 - year- old primigravid woman delivers a heafthy 3402 -g ( 7-lb 8 -oz) male newborn after an uncomplicated cesarean delivery because of a nonreassuring fetal stress test . Two days prior to discharge from the hospital she has persistent numbness of the
area surrounding the abdominal incision . The physician assures the patient that sensation will gradually return as the nerves regenerate. Which of the following best describes the rate -limiting step in this patient' s return to normal sensation?

. A ) Dorsal root ganglion cel! proliferation


8 ) Fast anterograde axonal transport
0 C ) Fibroblast proliferation
0 D ) Retrograde axonal transport
. E ) Slow anterograde axonal transport

Previous *
Next Lab Values Calculator
m
Review
O
Help
m
Pause
Exam Section 2: Item 5 of 50 National Board of Medical Examiners^ Time Remaining:
® Mark Comprehensive Basic Science Self-Assessment 1 hr 14 min 44 sec

5 . An 18-year -old woman comes to the physician because of progressive fever general malaise , and blood in her urine since she began oral antibiotic therapy for a urinary tract infection 5 days ago. She also has a 3- day history of a rash.. Her temperature is
38° C (1DG . 43F), pulse is 75/min, respirations are 12/ mrn, and blood pressure is 125/80 mm Hg. Physical examination shows a petechial rash over the chest . back : and upper and lower extremities. Urinalysis shows:
Blood 3+
Protein 1+
Leukocytes 150/hpf
Eosinophils 30%

Which of the following is the most likely diagnosis?

o A ) Acute tubular necrosis


6 } Glomerulonephritis
Q C ) IgA nephropathy
D ) Interstitial nephritis
O E ) Papillary necrosis

Previous *
Next Lab Values Calculator
m
Review
O
Help
m
Pause
Exam Section 2: Item 6 of 50 National Board of Medical Examiners^ Time Remaining:
9 Mark Comprehensive Basic Science Self-Assessment 1 hr 14 min 33 sec

6 . A 33- year- old woman has had weakness of the right lower two thirds of the face for the past 2 months . Which of the following labeled regions in the normal brain shown is the most likely site of the lesion causing this symptom?

Central sulcus
E F
C G
B /
\ r

> J
I

£$
(h :
kM
A5
-
.
r m-—
-
m wr
I--

r c d i- - Ll

J
-
«
3
» r.
j
i
.
H
^
ip
.

O A)
O B)
O C)
O D)
OE)
OF )
O G)
O H)
On
O J)

0
Previous
0
Next Lab Values Calculator
#
Review
0
Help
0
Pause
Exam Section 2: Item 7 of 50 National Board of Medical Examiners^ Time Remaining:
® Mark Comprehensive Basic Science Self-Assessment 1 hr 14 min 36 sec

7 . A 4 G -year- old woman comes to the physician because of temperatures of 38 to 39 X (100.4 to 102.2 aF) and malaise for the past 2 days. She has chronic alcohol dependence. There are several spider angiomas on the face , chest and back ; she is not
jaundiced . The abdomen is protuberant and nontender. There is shifting dullness to percussion . A firm liver edge is ballotable 3 cm below the right costal margin . Routine laboratory studies show miid anemia mildly increased hepatic transaminase activities :
;

and a decreased serum albumin concentration . Peritoneal aspiration yields serous fluid with 1100 leukocytes/mm 3 {80 neutrophils) and 100 erythrocytes / mm 3 Which of the following processes best accounts for the patient's febrife ilEness ?

A ) Acute cholecystitis
3 B ) Chronic pancreatitis
o C ) Exacerbation of autoimmune hepatitis
3 0 ) Pelvic infEammatory disease
E } Spontaneous bacterial peritonitis

O 9 0 0
Previous *
Next Lab Values Calculator Review Help Pause
Exam Section 2: Item 8 of 50 National Board of Medical Examiners^ Time Remaining:
9 Mark Comprehensive Basic Science Self-Assessment 1 hr 14 min 34 sec

8 . A patient with a 6-month history of heat intolerance. fatigue episodes of tachycardia and weight loss has a diffusely enlarged thyroid gland Serum concentrations of triiodothyronine (T 3) and thyroxine ( TJ are increased ; thyroid -stimulating hormone is
; :

decreased . Which of the following is the most likely diagnosis ?

O A ) Autoimmune thyroid hyperplasia


8 ) Pituitary neoplasm
O G ) Surreptitious ingestion of T 4
D ) Thyroid neoplasm

O
Previous *
Next Lab Values Calculator
m
Review
O
Help
m
Pause
Exam Section 2: Item 9 of 50 National Board of Medical Examiners^ Time Remaining:
9 Mark Comprehensive Basic Science Self-Assessment 1 hr 14 min 31 sec

9 . A 19- year- old man who is a college student is brought to the emergency department because of the sudden onset of right-sided chest pain and difficulty breathing after an accident in which he was thrown from his bicycle . He has difficulty walking and cannot
climb stairs because of pain and shortness of breath . He is slightly cyanotic, afebrile: and tachypneic . Which of the following is most suggestive that fractured ribs caused the respiratory problem?

A ) Bronchophony
B ) Expiratory stridor
0 C ) Inspiratory stridor
0 D ) Subcutaneous crepitus
. E ) Succussion splash

Previous *
Next Lab Values Calculator
m
Review
O
Help
m
Pause
Exam Section 2: Item 10 of 50 National Board of Medical Examiners^ Time Remaining:
® Mark Comprehensive Basic Science Self-Assessment 1 hr 14 min 29 sec

10 . A 62 -year- ofd woman develops difficulty breathing . Pulmonary function tests before and after branchediiator therapy show no changes . Predicted and patient values are :
Test Predicted Patient
FVC (L) 5.0 4.0
FEV 1 (L) 4.0 2.4
FEV /FVC 0.8 0.6
Total Eung capacity (L) 6.0 7.2
Residual volume (L) 1.6 2.7

Which of the following is the most likely explanation for these findings ?

Airway Resistance Lung Compliance


O A) T T
O B) T normal
O C) Normal 1
O 0) I T
O E) 4

Previous *
Next Lab Values Calculator
m
Review
O
Help
m
Pause
Exam Section 2: Item 11 of 50 National Board of Medical Examiners^ Time Remaining:
® Mark Comprehensive Basic Science Self-Assessment 1 hr 14 min 26 sec

11 . A 3-year- old girl is brought to the physician because of a 2 -week history of diarrhea . Her temperature is 37.6 CC ( 99.8 aF }: pulse is 70/min , respirations are Wmjn and blood pressure is 110/70 mm Hg . Physical examination shows generalized
;

[ ymphadenopathy. A CT scan of the chest and abdomen shows enlarged lymph nodes in the mesentery and para -aortic region. Examination of a lymph node biopsy specimen shows marked proliferation of histiocytes and numerous segmented neutrophils .
Granulomata are absent and special stains show numerous acid-fast bacilli. which are subsequently identified as Mycobacterium avium-intracellulare . Serum studies show normal concentrations of IgA, lgG IgM, B lymphocytes, T lymphocytes , and CD4? and
;

CD8 +- T lymphocytes. This patient most likely has defective function or expression of which of the following proteins?

A ) Class I MHG molecules


. 8 ) Interferon - gamma receptor
0 G ) interleukin 2 (IL 2) receptor
- -

) 0 ) Leukocyte function - associated antigen -1


'

E ) MADPH oxidase

Previous *
Next Lab Values Calculator
m
Review
O
Help
m
Pause
Exam Section 2: Item 12 of 50 National Board of Medical Examiners^ Time Remaining:
® Mark Comprehensive Basic Science Self-Assessment 1 hr 14 min 24 sec

12 Drug X is used to treat pain associated with rheumatoid arthritis . The drug is a weak acid with a pKa of 4.4; it is absorbed principally through the stomach . It is determined that Drug X is absorbed efficiently by the body because of the ionization conditions
,

under which it exists at gastric and blood pH . Which of the following sets of physical chemical states of the drug is most fikely?

At Gastric pH At Blood pH
O A) Ionized ionized
O B) Ionized nonionized
O C) Nonionized ioniized
O D) Nonionized nonionized

Previous *
Next Lab Values Calculator
m
Review
O
Help
m
Pause
Exam Section 2: Item 13 of 50 National Board of Medical Examiners^ Time Remaining:
® Mark Comprehensive Basic Science Self-Assessment 1 hr 14 min 21 sec

13. A previously healthy 10- year -old girl is brought to the physician by her mother because of a 6-week history of headache , nausea and difficulty walking . An MR ] of the brain shows a mass iin the posterior fossa that is found to be an astrocytoma i his tumor
, ,

developed from cells that normally serve which of the following functions in the brain?

. A ) Formation of myelin sheaths in the central nervous system


8 ) Phagocytosis of recycled synaptic terminal membrane
. C ) Production and secretion of cerebrospinal fluid
Q D ) Termination of action potentials
. E ) Transport of hormones from the cerebral spinal fluid to capillaries
F ) Uptake of amino acid neuretransmitters

Previous *
Next Lab Values Calculator
m
Review
O
Help
m
Pause
Exam Section 2: Item 14 of 50 National Board of Medical Examiners^ Time Remaining:
9 Mark Comprehensive Basic Science Self-Assessment 1 hr 14 min 19 sec

14. A 26 - year-old woman comes to the physician because of a 1 - week history of rectal pain that is made more severe by defecation, and occasional blood on the toilet tissue after a bowel movement . She says that her
stools appear normal and that she has not had any trauma . She has a history of chronic constipation. Visual rectal examination shows the findings in the photograph . Which of the following is the most likely diagnosis?

A ) Anal fissure
3 B ) Bowen carcinoma
C ) Condyloma acuminatum
O D ) Perianal abscess
E ) Prolapsed internai hemorrhoid

O & C
Previous *
Next Lab Values Calculator Review
*
Help Pause
Exam Section 2: Item 15 of 50 National Board of Medical Examiners^ Time Remaining:
9 Mark Comprehensive Basic Science Self-Assessment 1 hr 14 min 16 sec

15. A moderately obese 55-year -old man is brought to the emergency department because of a 10 -hour history of severe chest pain. He has a 5-year history of exercise-induced angina . His pulse is 109/min, respirations are 15/ min , and blood pressure is
132/92 mm Hg. Physical examination shows diaphoresis . A blood sample obtained 2 hours after admission shows increased serum activity of creatine kinase ME . Which of the following is the most likely cause of this aboratory finding?

. A ) Increased GoEgi complex activity


8 ) increased permeability of the plasma membrane
O G ) Mitochondria! swelling
D ) Muclear lysis
. E ) Proliferation of the endoplasmic reticulum

O
Previous *
Next Lab Values Calculator
9
Review
0
Help
0
Pause
Exam Section 2: Item 16 of 50 National Board of Medical Examiners^ Time Remaining:
9 Mark Comprehensive Basic Science Self-Assessment 1 hr 14 min 12 sec

16. AGG-year- ofd woman comes to the physician because of a 6 -month history of pain in her hips and knees. Physical examination shows findings consistent with osteoarthritis and the physician recommends iibuprofen . The patient refuses and asks about taking
;

glucosamine. Which of the following responses by the physician is most appropriate?

. A ) "Glucosamine hasn't been studied well enough for me to recommend it . "


8 ) "Glucosamine ' s side effects aren't listed . It may be more dangerous than we realize."
. G ) "Ibuprofen has been proven effective for your condition."
!
'

D ) "What have you heard about using glucosamine to treat arthritis?"


. E ) "Why did you come to me if you don’t want to take what I recommend? 11

F ) "You should really see a naturopathic doctor."

Previous *
Next Lab Values Calculator
m
Review
O
Help
m
Pause
Exam Section 2: Item 17 of 50 National Board of Medical Examiners^ Time Remaining:
9 Mark Comprehensive Basic Science Self-Assessment 1 hr 14 min 10 sec

17. An 88-year -old man who lives alone is brought to the physician by his daughter because she is concerned that he has not been eating a well -balanced diet for 9 months . He is 170 cm (5 ft 7 in) tall and weighs 50 kg (110 fb ); BMI is 17 kg/m 2 Physical
examination shows multiple ecchymoses on the upper and lower extremities . Laboratory studies show :
Platelet count 160,000/mm 3
Prothrombin time 12 sec ( INR =1 }
Serum
Vitamin B 6 ( pyridoxine) —
9 ng /dL (N= 5 30)
Vitamin C ( ascorbic acid ) 0.1 mg/dL (N=0.4-2)
Folic acid 5 ng/dL (N=2—20)

The ecchymoses in this patient are most likely due to a disorder of which of the following?

A ) Arachidonic add production


Q B ) Binding of carboxyglutamic acid to phospholipid
C C ) Carboxylation of factor fl (prothrombin)
J D ) Proline hydroxylafion
E } Transfer of methyl groups to organic acids

Previous *
Next Lab Values Calculator
m
Review
O
Help
m
Pause
Exam Section 2: Item 13 of 50 National Board of Medical Examiners^ Time Remaining :
® Mark Comprehensive Basic Science Self-Assessment 1 hr 14 min 3 sec

18. A 14 -year- ofd boy has persistent leukocytosis and neutrophilia without evidence of a current infection. He has a history of recurrent infections of the skin: upper and lower airways, and perirectal area . Gram-negative and gram-positive rods have been isolated .
The number and function of B and T lymphocytes are normal. Production of hypochlorous acid by neutrophils and the nitroblue tetrazolium reduction test are normal;. Neutrophil chemotactic response to the formyl -MetLeuPhe (fMLP) peptide is diminished .
Which of the following disorders of neutrophils is the most likely diagnosis?

A ) Chronic granulomatous disease


3 B ) Cyclic neutropenia
o C) Leukocyte adhesion deficiency
3 0 ) Myeloperoxidase deficiency
E } Neutrophil-specific granule deficiency

O
Previous *
Next Lab Values Calculator
m
Review
O
Help
m
Pause
Exam Section 2: Item 19 of 50 National Board of Medical Examiners^ Time Remaining :
® Mark Comprehensive Basic Science Self-Assessment 1 hr 14 min 5 sec

I 0-r0 1
x
2

n 1 1 > 4
^
0
1
2
3
£
3
2
3 6
^
2 3

HI
1
2
3
1
2
3
1 2
6
1
5
1
2
3
3
o
4

1 1 1
IV 2
3
2
3
5
6
1 2 3 4

Affected male
# Affected female
Unaffected male
O Unaffected female
0 Affected male , deceased
0 Unaffected female , deceased

19 . An investigator is studying a large family with many members who are affected by a disorder caused by a fully penetrant autosomal dominant inherited gene mutation. A pedigree is shown. Most affected members also have a rare allele at a focus thought to
be closely linked to the disease focus. A father (individual 111 -3) and his daughter ( individual iV- 3) have the disorder, but they have the wild-type allele at the linked locus . Which of the following is the most likely cause of these findings ?

( J A ) Insertion of a LINE sequence


8 } Random segregation
Q G ) Recombination
D ) Single nucleotide polymorphism
E ) Transduction

®
Previous
0
Next Lab Values Calculator
0
Review
0
Help
0
Pause
Exam Section 2: Item 20 ol 50 National Board of Medical Examiners^ Time Remaining :
® Mark Comprehensive Basic Science Self-Assessment 1 hr 14 min 3 sec

20 . A 45 -year- old woman comes to the physician because of a 6 -month history of shortness of breath with exeition and a nonproductive cough . She sometimes has difficulty swallowing and often has heartburn, especially if she fies down after a meal . She adds
that her fingers have swollen, and she has had to get her wedding ring resized . Her fingers also become white and painful in cold weather or cotd water. Her pulse is 75/min respirations are 20/min, and blood pressure is 150/100 mm Hg. Physical
:

examination shows tight, smooth facial skin without wrinkles . Additional testing is most likely to show which of the following sets of cardiovascular changes in this patient?

Left Ventricular Mean Pulmonary Coronary Vascular


Diastolic Compliance Artery Pressure Resistance
O A) f f f
O B) T T I
O C) T V
*

O D) 4 4
O E) v t t
O F) l '
i
O G) 1
A .

O H) 4 V V

Previous Next
fE
Lab Values Calculator
©
Review Help
15
Pause
Exam Section 2: Item 21 ol 50 National Board of Medical Examiners^ Time Remaining:
9 Mark Comprehensive Basic Science Self-Assessment 1 hr 13 min 45 sec

21 . A 25 -year- old woman comes to the physician for a routine health maintenance examination . She is currently preparing for an 8 -km (5-mile) race, and she sprint trains twice weekly. During this training : she runs 200-meter sprints in two groups of 10 sprints
separated by a 30-second rest between each sprint in a group: and a 2 -minute rest between each group of 10. After the workout her legs feel weak , and her muscles burn and sometimes cramp. Which of the following best explains her symptoms ?

A ) Decreased activity of the sodium - proton antiporter, resulting in an acidic sarcoplasm


8 ) increased activity of the sodium - proton antiporter resulting in an acidic sarcoplasm
O C ) increased oxygen delivery to the muscle : leading to increased metabolism and acid production
'

D ) Regeneration of NAD + from NADH which produces acid


;

Previous *
Next Lab Values Calculator
m
Review
O
Help
m
Pause
Exam Section 2: Item 22 ol 50 National Board of Medical Examiners^ Time Remaining:
® Mark Comprehensive Basic Science Self-Assessment 1 hr 13 min 42 sec

22 A 14 -year- old girl is brought to the physician because of a 1 -month history of migraine-like headaches, vomifing and multiple left-sided focal seizures . She has had hearing foss since the age of 11 years. Her mother and maternal grandmother have high -tone
:

deafness . Physical examination shows Joss of vision in one half of the visual field of the right eye and weakness of the right upper and tower extremities . Serum and cerebrospinal fluid concentrations of lactic acid are increased . This patient most likely has a
mutation of which of the fallowing?

O A ) Endoplasmic reticulum glycosyItransferase


3 B ) Lysosomal a- gfucosidase
o C ) Mitochondrial tRNALeu
3 0 ) Nuclear proteasome activator
J E } Peroxisomal catalase

O
Previous Next Lab Values Calculator
m
Review
O
Help
m
Pause
Exam Section 2: Item 23 of 50 National Board of Medical Examiners^ Time Remaining:
9 Mark Comprehensive Basic Science Self-Assessment 1 hr 13 min 40 sec

23. A case-control study is conducted to determine if obesity is a risk factor for gastroesophageal reflux disease ( GERD ). A questionnaire is used to select subjects with severe symptoms of GERD and subjects with no symptoms. A BMf is calculated for each
subject. The results ( in kg/ m 2) are shown:
BMI< 25 25 < BMK 30 30 < BMk 35 BMI> 35
Subjects with GERD symptoms 303 900 200 50
Subjects with no symptoms 700 900 200 30

Which of the following represents the odds ratio for GERD symptoms in subjects with BMfs greater than 35 compared with subjects with BMIs less than 25 ?

A ) (50 * 30) / (300 *700)


o B ) (50*700) I (30 300) x

o cj [50/(30+50)31 [300 (300+700)] /

: D ) [50/ (50+300)31 [ 30/(30+700)3


.
^ E ) [ 50/(50+200+900+300)3 [ 30/(30+200+900+700)3
'

Previous *
Next Lab Values Calculator
m
Review
O
Help
m
Pause
Exam Section 2: Item 24 ol 50 National Board of Medical Examiners^ Time Remaining:
0 Mar
*
Comprehensive Basic Science Self-Assessment 1 hr 13 min 33 sec

24. A 14 -year- old boy is brought to the physician by his mother because of daily headaches for 2 months . The headaches are described as a bilateral aching in the tempfes. His mother states that he also Tias not been himself ' for the past few months . He seems
more confused often forgetting names , dates , and places: and he is clumsy with frequent falls. His school performance also has declined over the past quarter. Physical examination shows a broad-based ataxic gait . He is alert and oriented to person: place ,
,
;

and time: but he is slow to answer questions . Chronic abuse of which of the following substances is the most likely cause of this patient' s condition?

A ) Cocaine
C B ) Ethanol
0 G ) Inhaled gfue
0 0 ) Methamphetamines
0 E } PCP (phencyclidine)

Previous *
Next Lab Values Calculator
m
Review
O
Help
m
Pause
Exam Section 2: Item 25 ol 50 National Board of Medical Examiners^ Time Remaining:
9 Mark Comprehensive Basic Science Self-Assessment 1 hr 13 min 36 sec

25. A study is designed to measure the impact of exercise on the incidence of myocardial infarction. Subjects are enrolled in the study and divided into two groups based on their self- reported exercise habits. At the end of the study subjects who reported
;

exercising have half the incidence of myocardial infarction compared with the subjects who did not exercise. Which of the following best describes this study design?

A ) Case- control
8 ) Case series
. C ) Cohort
o D ) Cross sectional
-

. E } Randomized clinical trial

Previous *
Next Lab Values Calculator
m
Review
O
Help
m
Pause
Exam Section 2: Item 26 ol 50 National Board of Medical Examiners^ Time Remaining:
9 Mark Comprehensive Basic Science Self-Assessment 1 hr 13 min 33 sec

26. An 18-year -old woman is being evaluated for amenorrhea . She has never had a menstrual period . She is 183 cm (6 ft) tall. Breast development and external genitalia are normal . There is no axillary or pubic hair. Which of the following karyotypes is most
likely?

O A ) 45.X
O 0 ) 46,xx
O C ) 46,XY
O D ) 46 X, i(Xq)
.

O E ) 47,XXX

Previous *
Next Lab Values Calculator
m
Review
O
Help
m
Pause
Exam Section 2: Item 27 of 50 National Board of Medical Examiners^ Time Remaining:
® Mark Comprehensive Basic Science Self-Assessment 1 hr 13 min 31 sec

27. A 50-year- old man comes to the physician because of a 2 - month history of pain of his wrists changes in skin color, and progressive fatigue. His brother has type 2 diabetes mefliitus and cirrhosis . Physical examination shows bronze- colored skin , tenderness
;

of the metacarpophalangeal joints in both hands, and hepatosplenomegaly. Serum studies show :
AST 100 U/ L
AIT 110 U /L
Ferritin 1200 ng/mL
Total iron-binding capacity 200 pg/dL (N=25 G-400)
Transferrin saturation 80% (N=20—50}

Analysis of a liver biopsy specimen shows a markedly increased iron concentration and cirrhosis . Which of the following is the most likely cause of the fmdings in this patient ?

A ) Increased erythropoietin action


Q B ) Increased intestinal iron absorption
Q C ) Increased oral iron intake
! D ) Decreased erythropoiesis
) E ) Decreased iron excretion
F ) Decreased serum transferrin concentration

Previous Next Lab Values Calculator


m
Review
O
Help
m
Pause
Exam Section 2: Item 28 ol 50 National Board of Medical Examiners^ Time Remaining:
® Mark Comprehensive Basic Science Self-Assessment 1 hr 13 min 29 sec

28. The presence of argininosuccinate in the urine indicates a defect in the conversion of which of the following?

0 A ) Ammonia to urea
J S ) Lysine to glutaryl CoA
0 G ) Methionine to succinyl CoA
J D ) Phenylalanine to fumarate
!J E ) Tryptophan to indole

Previous Next Lab Values Calculator


o
Review
o
Help
o
Pause
Exam Section 2: Item 29 of 50 National Board of Medical Examiners^ Time Remaining:
® Mark Comprehensive Basic Science Self-Assessment 1 hr 13 min 26 sec

29. A 55 - year- old woman is brought to the emergency department after being injured in a motor vehicle collision . She has an injury of the soft tissue of the face that prevents her from drinking from a glass without
spilling the contents. Several days later she stilt has this problem . The photograph shows her attempting to purse her lips to whistle. Which of the following nerves is most likely damaged?

A ) Buccal branch of the facial nerve


B ) Inferior alveolar branch of the mandibular division of the trigeminal nerve
O Infraorbital branch of the maxillary division of the trigeminal nerve
D ) Mandibular branch of the facial nerve
E ) Pharyngeal branches of the vagus nerve

Previous *
Next Lab Values Calculator
m
Review
O
Help
m
Pause
Exam Section 2: Item 30 ol 50 National Board of Medical Examiners^ Time Remaining:
® Mark Comprehensive Basic Science Self-Assessment 1 hr 13 min 24 sec

30 . A previously healthy 6-month- ofd boy is brought to the physician because of a cough for 1 week . Initially he had a low-grade fever sneezing congestion, and runny nose. He then developed a dry intermittent cough. The parents now note that with any startle
,

the baby chokes and gasps . He has not had any immunizations. Physical examination shows paroxysms of '‘machine gun" - like coughing with a forced expiratory grunt at the end of coughing . Leukocyte count is 30 000/mm 3 (70% lymphocytes ) . Neutrophil
;

chemotaxis and oxidative metabolism are defective due to increased activity of which of the following enzymes ?

A ) Adenyly! cyclase
3 B ) Myeloperoxidase
o C ) NADPH oxidase
O 0 ) Phospholipase C
J E } Protein kinase C

Previous *
Next Lab Values Calculator
m
Review
O
Help
m
Pause
Exam Section 2: Item 31 of 50 National Board of Medical Examiners^ Time Remaining:
® Mark Comprehensive Basic Science Self-Assessment 1 hr 13 min 22 sec

31. A 22 -year- old woman comes to the office because of a 3-day history of nonproductive cough. She also has a 1-week history of fatigue progressive shortness of breath with exertion and white lying down and swelling of her legs and feet . She delivered a male
:

newborn via uncomplicated vaginal delivery 1 month ago . She has no history of major medical illness and takes no medications . Her temperature is 37.7 aC ( 99.8 aF): pulse is 104/m in , respirations are 20/miinP and blood pressure is 126/80 mm Hg. Bilateral
basilar crackles are heard . There is 1+ edema of the lower extremities bilaterally. Which of the following is the most likely diagnosis ?

A ) Amniotic fluid embolism


3 B ) Cardiomyopathy
o C ) Major depressive disorder
3 0 ) Pneumonia
J E } Pulmonary embolism
' F ) Pulmonary fibrosis

o
Previous *
Next Lab Values Calculator
9
Review
0
Help
0
Pause
Exam Section 2: Item 32 ol 50 National Board of Medical Examiners^ Time Remaining:
® Mark Comprehensive Basic Science Self-Assessment 1 hr 13 min 20 sec

32 A 63-year- old man is scheduled to undergo coronary artery bypass grafting with a portion of the great saphenous vein. An incision to remove a portion of the vein should begin in which of the following locations ?

3 A ) Along the lateral surface of the leg


J S ) Along the medial side of the ankle joint
( 3 G ) Along the plantar surface of the foot
0 } Anterior to the knee joint
' J E ) Posterior to the hip joint

Previous *
Next Lab Values Calculator
m
Review
O
Help
m
Pause
Exam Section 2: Item 33 of 50 National Board of Medical Examiners^ Time Remaining:
9 Mark Comprehensive Basic Science Self-Assessment 1 hr 13 min 18 sec

33. A 62 -year- old man is brought to the emergency department 2 hours after the sudden onset of pain and coolness of his right leg . He is otherwise healthy except for mild hyperthyroidism treated with propylthiouracil . Examination of the tower extremities shows
normal skim naiis : and hair growth patterns . Pulses are absent in the right lower extremity and normal on the left . Which of the following is the most likely diagnosis ?

A ) Cellulitis
8 ) Deep venous thrombosis
O C) Embolic arterial occlusion
3 D ) Lumbar cisc herniation
. E } Rhabdomyolysis

Previous *
Next Lab Values Calculator
m
Review
O
Help
m
Pause
Exam Section 2: Item 34 ol 50 National Board of Medical Examiners^ Time Remaining:
9 Mark Comprehensive Basic Science Self-Assessment 1 hr 13 min 16 sec

34. A 32 -year- old woman comes to the physician for a follow - up examination after atypical cells were noted on a recent Pap smear. Physical examination shows a 1 x 1 -cm area of leukoplakia on the cervix A biopsy specimen of the lesion shows invasive
squamous cell carcinoma. Malignant cells from this site will most likely drain first to which of the following lymph nodes in this patient?

. A ) Femoral
8 ) inferior mesenteric
O C ) Internal iliac
D ) Lumbar
. E ) Superficial inguinal

Previous *
Next Lab Values Calculator
m
Review
O
Help
m
Pause
Exam Section 2: Item 35 ol 50 National Board of Medical Examiners^ Time Remaining:
9 Mark Comprehensive Basic Science Self-Assessment 1 hr 13 min 14 sec

35. A 70- year- old man dies in a motor vehicle collision . He had been undergoing evaluation for occult blood in the stool A photograph of a section of the transverse colon obtained at autopsy is shown. Which of the following is
the most likely diagnosis?

.. A ) Hyperplastic polyp
Q B ) Inflammatory pseudopolyp
C ) Juvenile polyp
O D ) Peutz-Jeghers syndrome
E ) Tubular adenoma

& * f*
Previous Next Lab Values Calculator Review Help Pause
Exam Section 2: Item 36 ol 50 National Board of Medical Examiners^ Time Remaining:
9 Mark Comprehensive Basic Science Self-Assessment 1 hr 13 min 12 sec

36. A 61-year- old man has erectile dysfunction due to spinal cord injury at L -2 . Sildenafil is likely to markedly correct the dysfunction by acting at which of the following labeled structures in the transverse section of the penis?

O A)
O B)
O C)
O Pi

0 0 ns © 0 0
Previous Next Lab Values Calculator Review Help Pause
Exam Section 2: Item 37 of 50 National Board of Medical Examiners^ Time Remaining:
9 Mark Comprehensive Basic Science Self-Assessment 1 hr 13 min 10 sec

40-i
37. Which of the following is the mean number of episodes of urinary tract infections for children (n= 100 } in the sample shown in the graph?

O A) 1 g 30- I

c B ) 1.55 J iBUBn

o
o C ) 2.07 8 2 -
°
D ) Cannot be determined from this graph _Q

I 10- «

0 1 2 3
Number of urinary tract
infections

Previous Next Lab Values Calculator


m
Review
O
Help
m
Pause
Exam Section 2: Item 33 ol 50 National Board of Medical Examiners^ Time Remaining :
® Mark Comprehensive Basic Science Self-Assessment 1 hr 13 min 3 sec

38. A 3-year- old boy is brought to the physician because of a 1- month history of pale skin . His parents are of European descent . He has no personal or family history of major medical illness. Physical
examination shows pallor. Laboratory studies show :
Hemoglobin 3 g.' dL (N=11-15 )
Hematocrit 24% (N= 28-45)
Mean corpuscular hemoglobin concentration 34% Hb/cell ( IM=31-36)
Mean corpuscular volume 90 pm 3 (N=77-98)

A photomicrograph of a peripheral blood smear is shown . Genetic testing is most likely to show which of the following findings in this patient?

A ) Heterozygous mutation in the ankyrin gene


B ) Heterozygous mutation in the a-globin gene
O P ) Heterozygous mutation in the p-giobin gene
-

D ) Homozygous mutation in the ankyrin gene


O E ) Homozygous mutation in the a- globin gene
F ) Homozygous mutation in the P- globin gene

Previous *
Next Lab Values Calculator
m
Review
O
Help
m
Pause
Exam Section 2: Item 39 of 50 National Board of Medical Examiners^ Time Remaining :
® Mark Comprehensive Basic Science Self-Assessment 1 hr 13 min 5 sec

39. A 24 -year- old woman is brought to the emergency department 15 minutes after the sudden onset of shortness of breath following a bee sting . Her pulse is 130/min , and blood pressure is 70/ 30 mm Hg . Three hours later, her blood pressure returns to normal
following administration of intravenous fluids corticosteroids . antihistamines: and epinephrine . The next day. she has minimal urine output . Which of the following areas of the kidney is likely to be most affected with this patient's prolonged hypotension?
,

O A ) Glomerular epithelial cells


8 ) Loop of Henle
0 G ) Medullary interstifium
D ) Mesangial cells
. E ) Proximal tubules

Previous *
Next Lab Values Calculator
m
Review
O
Help
m
Pause
Exam Section 2: Item 40 ol 50 National Board of Medical Examiners^ Time Remaining :
® Mark Comprehensive Basic Science Self-Assessment 1 hr 13 min 3 sec

40 . A 65 -year- old man develops urinary incontinence immediately after an operation for prostate cancer. The most likely cause of his condition is damage to the prostatic nerve plexus that resulted in denervation of the internal urethrai sphincter. The function of
which of the following types of tissue is most likely impaired as a result of this damage ?

. A ) Dense irregular connective tissue


8 ) Dense regular connective tissue
. C ) Skeletal muscle
D ) Smooth muscle
. E } Transitional epithelium

Previous *
Next Lab Values Calculator
m
Review
O
Help
m
Pause
Exam Section 2: Item 41 ol 50 National Board of Medical Examiners^ Time Remaining :
® Mark Comprehensive Basic Science Self-Assessment 1 hr 13 min 0 sec

41 . A 10-year- old boy who has begun chemotherapy for acute myelogenous leukemia awakens at night with fever and severe pain in the ankles . Treatment with over -the -counter analgesics does not resolve the pain . The next morning , he has pain with urination
and blood in the urine . An increased serum concentration of which of the following compounds is the most likely cause of this patient's symptoms?

A ) Cystine
8 ) Glycine
. ; C ) Magnesium
V D ) Urea
. . E ) Uric acid

Previous *
Next Lab Values Calculator
m
Review
O
Help
m
Pause
Exam Section 2: Item 42 ol 50 National Board of Medical Examiners^ Time Remaining:
9 Mark Comprehensive Basic Science Self-Assessment 1 hr 12 min 53 sec

42 A 27-year- old woman is brought to the physician by her family because she has been progressively lethargic and unwilling to leave her apartment over the past week . She has been receiving treatment in a mental health center for 10 years but missed her last
appointment 13 days ago because of a snowstorm. She is now reluctant to return because she believes the staff is involved in an extraterrestrial plot . Three years ago she had similar symptoms treated with electroconvulsive therapy. She appears
;

disheveled . She is having auditory hallucinations of several people talking about her. Physical examination shows normal findings. She has poor eye contact, a flat affect and slow speech. She describes an elaborate delusional system about the plot at the
mental health center. Thought content is otherwise impoverished. Which of the following is the most likely diagnosis?

A ) Bipolar disorder depressed


. B ) Borderline personality disorder
0 C ) Delusional disorder
) D ) Schizophrenia
E ) Schizotypal personality disorder

Previous *
Next Lab Values Calculator
m
Review
O
Help
m
Pause
Exam Section 2: Item 43 of 50 National Board of Medical Examiners^ Time Remaining:
® Mark Comprehensive Basic Science Self-Assessment 1 hr 12 min 56 sec

43. A viral gene product is found to decrease expression of class i MHC molecules on the surfaces of infected cells. A mutant strain of virus is isolated that has a nonfunctional form of this gene . Which of the following types of cells are likely to contribute more
effectively to control of the parental strain of the virus than to control of the mutant virus ?

0 A ) CD 4 +- 7 lymphocytes
8 ) CD8 +- T lymphocytes
. ) C ) Follicular dendritic cells
'

_ D ) Natural killer cells


. . E } Plasma cells

Previous *
Next Lab Values Calculator
m
Review
O
Help
m
Pause
Exam Section 2: Item 44 ol 50 National Board of Medical Examiners^ Time Remaining:
® Mark Comprehensive Basic Science Self-Assessment 1 hr 12 min 54 sec

44. A previously healthy 40- year -old man is brought to the emergency department 1 hour after the sudden onset of severe pain in his left leg while playing tennis. He is found to have ruptured the left Achilles tendon and undergoes operative repair and long leg
cast immobilization . Six months later the left calf shows a 2-cm decrease in circumference compared with the right calf . Which of the following is the most likely cause of this decrease?
,

. A ) Decreased glycogen synthesis


8 ) Decreased myosin light chain phosphatase activity
. J C ) Increased phosphatidyl degradation
'

j D ) increased protein degradation


. E ) Mitochondria damage
F ) Necrosis of muscle fibers

Previous *
Next Lab Values Calculator
m
Review
O
Help
m
Pause
Exam Section 2: Item 45 ol 50 National Board of Medical Examiners^ Time Remaining:
® Mark Comprehensive Basic Science Self-Assessment 1 hr 12 min 50 sec

45. A 5 G - year- old woman has recently diagnosed carcinoma of the breast. An x -ray of the chest shows a tumor next to the right side of the heart. An enhanced CT scan with the tumor invading the
pericardium is shown. Which of the following structures is most likely involved?

A ) Coronary sinus
B ) Greater splanchnic vein
C ) Right phrenic nerve
D ) Right vagus nerve
E ) Thoracic duct

O
Previous *
Next Lab Values Calculator
9
Review
0
Help
0
Pause
Exam Section 2: Item 46 ol 50 National Board of Medical Examiners^ Time Remaining:
® Mark Comprehensive Basic Science Self-Assessment 1 hr 12 min 4£ sec

46. A 25 - year- old woman with a history of rheumatic fever and mitral valve dysfunction comes to the physician because of a 2 - week history of fever and fatigue. She underwent a root canal procedure 1 month ago ;

before which she had taken a single dose of amoxicillin. Her temperature is 38.4 * 0 (101 29F). A grade 4 /6 blowing murmur is heard on auscultation under the left axilla . A photomicrograph of a Gram stain of the
organism recovered from a blood culture specimen is shown . On blood agar plates the organism shows alpha hemolysis . Which of the following is the most likely causal organism?
;

A ) Enterococcus faecalis
B ) Group A beta-hemolytic streptococci
0 C ) Staphylococcus aureus
D ) Streptococcus mitts
E ) Streptococcus pneumoniae

O
Previous *
Next Lab Values Calculator
m
Review
O
Help
m
Pause
Exam Section 2: Item 47 ol 50 National Board of Medical Examiners^ Time Remaining:
® Mark Comprehensive Basic Science Self-Assessment 1 hr 12 min 46 sec

47. A 24 -year- old woman comes to the physician for advice about contraceptive methods . She is recently married and is not interested in having children until her mid 30 s . Which of the following contraceptives carries the highest risk for interference with fertility in
this patient Eater in life ?

. A ) Cervical cap with spermicidal jelly


8 ) Condoms and spermicidal foam
o C ) JUD
o D ) Oral contraceptive
. . E ) Progestin implant

O
Previous Next Lab Values Calculator
m
Review
O
Help
m
Pause
Exam Section 2: Item 43 ol 50 National Board of Medical Examiners^ Time Remaining:
® Mark Comprehensive Basic Science Self-Assessment 1 hr 12 min 44 sec

48. Topical corticosteroid creams and phototherapy are often effective in the treatment of psoriasis. This effectiveness suggests a role for metabolism or nuclear binding of which of the following vitamins in the treatment of psoriatic lesions ?

A ) Niacin

^0 G S ) Vitamin B 1 (thiamine)
) Vitamin B 2 (riboflavin)

J 0 } Vitamin B 6 (pyridoxine )
0 E ) Vitamin C
'
. F ) Vitamin D
0 G ) Vitamin E
H } Vitamin K

Previous *
Next Lab Values Calculator
m
Review
O
Help
m
Pause
Exam Section 2: Item 49 of 50 National Board of Medical Examiners^ Time Remaining:
® Mark Comprehensive Basic Science Self-Assessment 1 hr 12 min 42 sec

49. A 21-year- old woman of Japanese descent comes to the emergency department because of a 3-hour history of facial flushing . Her symptoms began shortly after she drank a glass of champagne for the first time at a wedding reception . Physical examination
shows profound erythema over the face . The most likely cause of these findings is a mutation in the gene for which of the following enzymes?

. A ) Acetyl- CoAreductase
8 ) Alcohol catalase
. ) C ) Alcohol dehydrogenase
'

_ D ) Alcohol reductase
. E ) Aldehyde dehydrogenase

O
Previous *
Next Lab Values Calculator
9
Review
0
Help
0
Pause
Exam Section 2: Item 50 ol 50 National Board of Medical Examiners^ Time Remaining:
® Mark Comprehensive Basic Science Self-Assessment 1 hr 12 min 39 sec

50 . A 35 -year- old woman is brought to the emergency department after she sustains a fracture of the neck of the fibula of her right leg . She was struck by a car while crossing the street . Which of the following findings is most likely on examination of the affected
leg?

Pain Over Pain Over


Proximal Fibula Distal Fibula Dorsiflexion Plantar Flexion Achilles Reflex
O A) Absent absent 4/ 5 1/ 5 1+
O B) Absent present 0/S 4/ 5 1+
O C) Present absent 1/ 5 4/ 5 2+
O D) Present absent 5/ 5 0/ 5 absent
O E) Present present 4/ 5 4/ 5 2+

Previous *
Next Lab Values Calculator
m
Review
O
Help
m
Pause
Exam Section 3: Item 1 of 50 National Board of Medical Examiners^ Time Remaining:
9 Mark Comprehensive Basic Science Self-Assessment 1 hr 14 min 57 sec

1. AG5 - year- old man comes to the physician because of fever and a worsening cough productive of approximatefy % cup of blood-tinged sputum daily. He has smoked VA packs of cigarettes daily for 45 years . His temperature is 38.4X (101 2 aF) respirations
;

are 20/min, and blood pressure is 140/90 mm Hg . Physical examination shows bilateral clubbing of the digits. Pulmonary examination shows egophony. whispered pedoriloquy, and dullness to percussion in the area overlying the 2nd and 3rd ribs on the right
anteriorly just inferior to the right clavicle. Which of the following structures is the most likely site of an obstructing carcinoma?

A ) Main carina
0 B ) Right lower lobe bronchus
0 G ) Right main bronchus
0 0 ) Right middle lobe bronchus
J E } Right upper lobe bronchus

Next Lab Values Calculator


m
Review
O
Help
m
Pause
Exam Section 3: Item 2 of 50 National Board of Medical Examiners^ Time Remaining:
9 Mark Comprehensive Basic Science Self-Assessment 1 hr 14 min 54 sec

2 . A 43- year- old woman comes to the physician because of a 4 - week history of intermittent episodes of severe abdominal pain : especially after eating fatty foods . Physical examination shows no abnormalities . Abdominal ultrasonography shows several
3 - to 4-mm gallstones contained within the gallbladder. The patient declines surgery. A drug with which of the following mechanisms of action is most appropriate for this patient?

. A ) Decreased gallbladder muscle contractility


8 ) increased gastrin production
. J C ) increased metabolism of bite salts
o D ) inhibition of biliary cholesterol secretion
. E } Inhibition of cytochrome P 450 activity

O
Previous Next Lab Values Calculator
m
Review
O
Help
m
Pause
Exam Section 3: Item 3 of 50 National Board of Medical Examiners^ Time Remaining:
® Mark Comprehensive Basic Science Self-Assessment 1 hr 14 min 52 sec

3 . A 24 - year- old woman comes to the physician for an examination prior to employment . Physical examination shows no abnormalities . An ECG shows a sinus rhythm of 70/ min PR interval of 152 msec and QRS complex of 84 msec : there are no ectopic beats.
;

Which of the following best represents the longest conduction time in this patient?

A ) Anterior left bundle branch


8 ) intra -atrial
G ) Low right atrium to bundle of His
D ) Proximal bundle of His to ventricular myocardium
E ) Right bundle branch

O
Previous *
Next Lab Values Calculator
m
Review
O
Help
m
Pause
Exam Section 3: Item 4 of 50 National Board of Medical Examiners® Time Remaining:
® Mark Comprehensive Basic Science Self-Assessment 1 hr 14 min 46 sec

4 . An 18-year -old woman with mild intellectual disability is brought to the physician because of a 3-day history of decreased ability to see in reduced light . She has a lifelong history of chronic diarrhea . Two years ago she developed a lack of muscle control of her
;

arms and legs: and generalized weakness. Her 16-year -old brother has had similar symptoms. Ophthalmologic examination shows bilateral retinitis pigmentosa . There is ataxia and loss of deep tendon reflexes . Laboratory studies show erythrocytes with spiny
projections and a serum total cholesterol concentration of 40 mg/dL . Which of the following apolipoproteirs is most likely deficient in this patient?

O A ) ApoA-l
O B ) Apo A-l
o C } Apo B
O D } Apo C
O E ) Apo E

o
Previous
o
Next Lab Values Calculator
&
Review
r
Help
#
Pause
Exam Section 3: Item 5 of 50 National Board of Medical Examiners^ Time Remaining:
® Mark Comprehensive Basic Science Self-Assessment 1 hr 14 min 41 sec

5 . An investigator is studying obesity in an adolescent population. Five thousand normal-weight patients are followed from the age of 10 years to the age of 15 years . At the conclusion of the study, 1100 patients meet the criteria for obesity. Which of the following
best represents the incidence per 1000 patient-years of obesity during the course of this study ?

O A ) 10
O 8 ) 25
O C ) 44
o D ) 50
O E) 64

O
Previous *
Next Lab Values Calculator
m
Review
O
Help
m
Pause
Exam Section 3: Item 6 of 50 National Board of Medical Examiners^ Time Remaining:
9 Mark Comprehensive Basic Science Self-Assessment 1 hr 14 min 33 sec

t "

*y?vrG>H
u i

- ^- -v ^ _
A d c* a£F £
6 . A 2 S -year- old woman at 32 weeks ' gestation comes to the physician because of a 4 -day history of fever and back pain . She says :hat during this time she afso has been crying frequentfy. Her temperature is 38 X v\ 4 A
* _

* . >v - *
i
i
. . *
Hi
I

(100.4°F) . Physical examination shows costophrenic angle tenderness. The photomicrograph shown represents her disease. Which of the following mechanisms iis the most likely cause? I
i
T' a a sf i ^ |
- .*i L

* J
\ F

» i:• *1 * V I
*
i
- *j*' •*' ' *>t** * * %

Q
f

A ) Chorioamnionitis
* V- -
¥
S'L
3 B ) Endometritis * V .
41
P “
k * !r

n
r ~
- Sr -i
i
li

-
*-
n

Q G ) Glomerulonephritis ill r

j
;&3r
4
• .
'
L *
S... ,
ii

O D ) Hematogenous infection I
. *
1
rrn * •
-r
i

' >
r
PL U
t /
Q E } Obstructive uropathy t ' -
*
I
*»*

F ) Pelvic infEammatory disease


.:
0%
l /^
v -I
.
»

M ! *
\ A- * h *
,vF rf I
1* -1
l* § + *
I
wf
--
1
K
-;
- .’ - F
JI
'V


V l! r'rt s

**
twit|V H W ,
%» ,1
w
nJ

* *
m
f
:

. 1!4

©
Previous Next Lab Values Calculator
©
Review Help Pause
Exam Section 3: Item 7 of 50 National Board of Medical Examiners^ Time Remaining:
® Mark Comprehensive Basic Science Self-Assessment 1 hr 14 min 35 sec

7 . A 54 -year- old man with a 1-year history of multiple sclerosis has increasingly painful muscle spasms . An agonist that acts at which of the following receptors is most likely to alleviate the increased extensor tone and clonus ?

3 A ) y - Aminobutyricacid E

^o G S ) Histamine -1 (Ht)
) Histamine -2 (H
i D ) Muscarinic- 1 (M )
^,
3 E ) Muscarinic- 2
3 F ) Nicotinic

o
Previous Next Lab Values Calculator
m
Review
O
Help
m
Pause
Exam Section 3: Item 8 of 50 National Board of Medical Examiners^ Time Remaining:
® Mark Comprehensive Basic Science Self-Assessment 1 hr 14 min 32 sec

8 . A 27 - year- old primigravid woman at 36 weeks ' gestation comes to the physician for a prenatal visit . Physical examination shows a uterus consistent in size with a 33- week gestation , so the patient is referred for ultrasonography, which shows normal fetal
measurements. The ultrasonography showrn is an image of the male fetal scrotum; the testes are indicated by the arrows . Which of the following is the most likely underlying cause of this finding?

A } High attachment of the tunica albuginea


. 8 ) Lack of scrotal attachment of the guberrtaculum
'

G ) Obstruction of the posterior urethra


J 0 ) Patent processus vaginalis
E } Twisting of the spermatic cord

Previous *
Next Lab Values Calculator
m
Review
O
Help
m
Pause
Exam Section 3: Item 9 of 50 National Board of Medical Examiners^ Time Remaining:
® Mark Comprehensive Basic Science Self-Assessment 1 hr 14 min 28 sec

9 . A 98 - year- old woman dies of respiratory failure. At autopsy: gross examination shows a small heart with a brown discoloration. Histologic examination of the cardiac tissue by light microscopy shows fight brown pigment within the cardiac myocytes particularly
,

in the perinuclear region . This material does not stain positively for iron . Which of the following best describes this pigment?

. A ) Anthracotic pigment
8 ) Free fatty acids
. C ) Hemosiderin
D ) Laminin
. . E ) Lipofuscin

O
Previous *
Next Lab Values Calculator
m
Review
O
Help
m
Pause
Exam Section 3: Item 10 ol 50 National Board of Medical Examiners^ Time Remaining:
® Mark Comprehensive Basic Science Self-Assessment 1 hr 14 min 24 sec

10 . A 43-year- old woman has had increasing fatigue intolerance to cold and dry thickened skin for 7 weeks . The thyroid gland is firm, nodular and diffusely enlarged . Examination of tissue obtained on biopsy of the thyroid gland shows a diffuse infiltration of
,
:

lymphocytes with occasional lymphoid follicles . Which of the following is the most likely diagnosis ?

. A ) Chronic autoimmune (Hashimoto) thyroiditis


8 ) Diffuse toxic goiter (Graves disease )
0 C ) Granulomatous thyroiditis
0 D ) Papillary carcinoma
. E ) Toxic multinodular goiter
F ) Viral thyroiditis

Previous *
Next Lab Values Calculator
m
Review
O
Help
m
Pause
Exam Section 3: Item 11 of 50 National Board of Medical Examiners^ Time Remaining:
® Mark Comprehensive Basic Science Self-Assessment 1 hr 14 min 20 sec

11 . A 3-year- old girl has a history of recurrent infections fn vitro neutrophils isolated from this patient are capable of phagocytosis and can kill Lactobacillus species but not Staphylococcus aureus. This patient most likely has a defect involving which of the
, ,

following enzymes ?

. A ) Catalase
8 ) Elastase
. C ) Myeloperoxidase
o D ) NADPH oxidase
. E } Superoxide dismutase

O
Previous Next Lab Values Calculator
m
Review
O
Help
m
Pause
Exam Section 3: Item 12 ol 50 National Board of Medical Examiners^ Time Remaining:
9 Mark Comprehensive Basic Science Self-Assessment 1 hr 14 min 17 sec

12 A 7 - year- old girl is brought to the physician because of a 2-week history of painful swelling under her right ami which has become increasingly severe during the past 3 days . Physical examination shows a 2 x 1- cm , tender right axillary lymph node and a
small papule on the dorsum of the right hand . A silver -stained biopsy specimen of the papule shows pleomorphic bacilli. Which of the following is the most likely causal organism?

O A } Bartonella henselae
8 ) Brucella melitensis
o C) Burkholder!a mallei
0 D ) Franciselta tuiarensis
. E ) Streptobacitlus moniliformis

Previous *
Next Lab Values Calculator
m
Review
O
Help
m
Pause
Exam Section 3: Item 13 of 50 National Board of Medical Examiners^ Time Remaining:
9 Mark Comprehensive Basic Science Self-Assessment 1 hr 14 min 14 sec

13. A newborn born at 3u weeks ' gestation dies 3 days later The photograph shown is of a section of brain as seen at autopsy. Which of the following is the most likely underlying disease?

A ) Biliary atresia
B ) Cyanotic congenital heart disease
O ® ) Hemolytic disease of the newborn
D ) Neonatal meningitis
O E ) Respiratory distress syndrome

Previous Next Lab Values Calculator


o
Review
o
Help
o
Pause
Exam Section 3: Item 14 of 50 National Board of Medical Examiners^ Time Remaining:
9 Mark Comprehensive Basic Science Self-Assessment 1 hr 14 min 10 sec

14 . A 50-year- ofd woman is brought to the emergency department 20 minutes after the sudden onset of pain in her epigastrium that radiates to her back and is associated with nausea . She has a history of cholelithiasis . She appears restless. Her temperature is
37.6X ( 99.7 aF) : pulse is 120/min , and btood pressure is 115/60 mm Hg . Abdominal examination shows distention with rebound tenderness and guarding. The pain is partially relieved when the patient bends forward . ACT scan of the abdomen shows fluid
surrounding the pancreas. Obstruction of which of the following is the most likely cause of the findings in this patient?

A ) Ampulla of Vater
0 B ) Cisterna chyli
0 G ) Common hepatic duct
0 0 ) Cystic duct
J E } Superior mesenteric artery

Previous *
Next Lab Values Calculator
m
Review
O
Help
m
Pause
Exam Section 3: Item 15 ol 50 National Board of Medical Examiners^ Time Remaining :
® Mark Comprehensive Basic Science Self-Assessment 1 hr 14 min 7 sec

15. A 74 -year- ofd man comes to the office because of a 3-week history of progressive shortness of breath with exertion. He has had difficulty climbing one flight of stairs. He also has a 20-year history of poorly controlled hypertension, for which he rarely takes his
prescribed medication and osteoarthritis treated with naproxen daily. His temperature is 37.1CC ( 98.8 QF) puise is 76/min , respira:ions are 20/min, and blood pressure is 180/ 98 mm Hg . Pulse oximetry on room air shows an oxygen saturation of 92% . Physical
;

examination shows 10- cm jugular venous distention above the sternal angle . Crackies are heard over both lung bases. An S 4 is heard . Echocardiography shows a normal ejection fraction . Which of the following best explains this patient' s symptom?

) A ) Decreased myocardiaE oxygen supply


' J B ) External compression of the right ventricle
0 C ) Impaired left ventricular relaxation
' J D ) Increased turbulent flow across the mitral valve

J E } Obstruction of the left ventricular outflow tract

O 0 I* 0 0 0
Previous Next Lab Values Calculator Review Help Pause
Exam Section 3: Item 16 ol 50 National Board of Medical Examiners^ Time Remaining :
® Mark Comprehensive Basic Science Self-Assessment 1 hr 14 min 4 sec

16. A study is conducted to assess the effect of a new selective estrogen receptor modulator (SERM ) on bone fractures . Two thousand women with a history of vertebral fracture and 5000 women with no 25 i
fracture history are randomly assigned to a placebo or to SERM at a dose of 60 mg or 120 mg . The graph shows the percent of women who had a new fracture during 5 years of foJIow-up T
(± standard error of the mean). Which of the following is the most accurate conclusion about preventing new fractures based on these data ?
£ 20
A ) High-dose SERM treatment is more effective than low- dose SERM treatment o
T T
B ) Low - dose SERM treatment is more effective than high -dose SERM treatment 15
3 C ) Placebo treatment is effective only in women with a history of vertebral fracture
0)
D ) SERM treatment is effective only in women with a history of vertebral fracture 10
E ) SERM treatment is effective only in women with no history of fracture
£o
5 - T T T

0
No fracture history Fracture history

Placebo
SERM 60 mg
SERM 120 mg

O 9 0 0
Previous *
Next Lab Values Calculator Review Help Pause
Exam Section 3: Item 17 of 50 National Board of Medical Examiners^ Time Remaining :
® Mark Comprehensive Basic Science Self-Assessment 1 hr 14 min 0 sec

17. A 23-year- ofd man cuts his tip inadvertently while shaving. Seconds after the injury the bleeding nearly stops. Which of the following mechanisms is the most likely cause of the early rapid control of blood loss in this man?

3 A ) Activation of antithrombin Hi

^ 8 } Localized secretion of endothelin


Q G ) Oxygen stimulated cleavage of thromboplastin
-

D ) Polymerization of fibrin

«r
Previous *
Next Lab Values Calculator
m
Review
O
Help
m
Pause
Exam Section 3: Item 13 ol 50 National Board of Medical Examiners^ Time Remaining:
9 Mark Comprehensive Basic Science Self-Assessment 1 hr 13 min 57 sec

18. A 21-year- old woman comes to the physician for a routine health maintenance examination . Her 3-year- old sister was recently diagnosed with cystic fibrosis . Prior to any genetic testing which of the following best approximates this patient' s risk of being a
;

heterozygote for cystic fibrosis?

O A) 0
O 8 ) 1/2
O C ) 1/ 4
o D ) 2/3
O E } 3/4

O
Previous *
Next Lab Values Calculator
9
Review
0
Help
0
Pause
Exam Section 3: Item 19 of 50 National Board of Medical Examiners^ Time Remaining:
9 Mark Comprehensive Basic Science Self-Assessment 1 hr 13 min 54 sec

19. A 25 -year- old woman with a 3-year history of celiac disease comes to the physician because of a markedly itchy rash on her scalp arms , buttocks: and legs for 5 weeks. Examination of the skin shows many crusted papules surrounded by excoriations over
,

the upper and lower extremities and a few vesicles on the buttocks . A skin biopsy specimen shows the vesicles to be subepidermal . Direct immunofluorescence of the vesicles shows granular deposits of IgA in the epidermal-dermal junction . Which of the
following is the most likely diagnosis ?

A ) Bullous pemphigoid
3 B ) Dermatitis herpetiformis
o G ) Dyshidrotic eczema
3 D ) Epidermolysis bullosa acquisita
J E ) Pemphigus

Previous *
Next Lab Values Calculator
m
Review
O
Help
m
Pause
Exam Section 3: Item 20 ol 50 National Board of Medical Examiners^ Time Remaining:
9 Mark Comprehensive Basic Science Self-Assessment 1 hr 13 min 50 sec

20 . A 1 - year- old boy is brought to the physician for a weli- chiid examination . The mother is concerned because her son' s fine , pale hair has not changed color since birth . His eyes are blue . During ophthalmologic examination the patient turns away from the
,

flashlight and starts crying . Which of the following is the most likely cause of the paie skin color in this patient ?

. A ) Aberrant migration of neural crest cells


8 ) Decreased number of epidermal melanocytes
. J C ) immune destruction of melanocytes
3 D ) inability to produce melanin
'

. E } Melanin dropout to the dermis

O
Previous *
Next Lab Values Calculator
m
Review
O
Help
m
Pause
Exam Section 3: Item 21 ol 50 National Board of Medical Examiners^ Time Remaining:
® Mark Comprehensive Basic Science Self-Assessment 1 hr 13 min 46 sec

21 . A GS -year- old man with recurrent urinary tract infections comes to the physician because of a 2-day history of fever and pain with urination. Urinalysis shows pus . He is treated with fevofloxacin. His symptoms worsen , and a blood culture grows Pseudomonas
aeruginosa that is resistant to levofloxacin . Alteration of which of the following best explains the resistance of P. aeruginosa to levofloxacin in this patient?

O A ) Dihydrofolate reductase
8 ) p - Lactamase
. J C ) 30S ribosomal subunit
D ) Topoisomerase
. E } Transpeptidase

Previous *
Next Lab Values Calculator
o
Review
o
Help
o
Pause
Exam Section 3: Item 22 ol 50 National Board of Medical Examiners^ Time Remaining:
® Mark Comprehensive Basic Science Self-Assessment 1 hr 13 min 42 sec

22 A sexually active 20-year -old woman has had painful, erythematous , vesicular lesions on the vulva for 2 days. Examination by speculum also shows lesions in the vagina! vault and on the cervix . A photomicrograph of a scraping of the base of an unroofed
vesicle is shown. Which of the following is the most likely diagnosis ?

A ) Candidal vaginitis
B ) Condylomata acuminata
. C ) Genital herpes
O D } SyphiEis
! _ E ) Trichomonas vaginalis vaginitis

O
Previous *
Next Lab Values Calculator
m
Review
O
Help
m
Pause
Exam Section 3: Item 23 of 50 National Board of Medical Examiners^ Time Remaining:
9 Mark Comprehensive Basic Science Self-Assessment 1 hr 13 min 39 sec

23. A 67-year- old woman with Crohn disease is admitted to the hospital for bowel rest and parenteral hyperalimentation. A central venous catheter is placed. Seven days after admission , she develops a temperature of 39i2*C (102.63F) and shaking chills . Broad -
spectrum antibiotics are administered pending blood cultures . She continues to have daily temperature spikes. Two days later blood culture grows yeast forms. Which of the following is the most likely causal organism?

O A ) Aspergillus turnig atus


B ) Blastomyces dermatitidls
0 C ) Candida albicans
0 D ) Cr/ ptococcus neoformans
. E ) Mucorramosissimus

Previous *
Next Lab Values Calculator
m
Review
O
Help
m
Pause
Exam Section 3: Item 24 ol 50 National Board of Medical Examiners^ Time Remaining:
0 Mar
*
Comprehensive Basic Science Self-Assessment 1 hr 13 min 35 sec

24. A 25 -year- old woman comes to the physician because of a 6 -month history of a white discharge from her breasts . Her fast menstruaf period was 6 months ago. She has had no headaches or visual field changes . She has never been sexualiy active. Gentle
palpation of both breasts produces expression of a milky discharge . Serum studies show :
Thyroid-stimulating hormone 1.5 |jU/mL
Follicle-stimulating hormone 6 mlLI / mL
Luteinizing hormone 6 mlU /mL
Prolactin 200 ng/mL

A 5 -day course of medroxyprogesterone fails to induce menses . An MRI of the brain shows an 8-mm ; esion in the pituitary gland . The most appropriate treatment for this patient is a drug from which of the following classes ?

3 A ) Dopamine agonist
_ . 6 } Dopamine antagonist
Q C ) Oxytocin agonist
D ) Oxytocin antagonist
,
^ F ) Serotonin (5 HT ,) antagonist
E ) Serotonin (5-HT ) agonist
-

Previous *
Next Lab Values Calculator
0
Review
G
Help
®
Pause
Exam Section 3: Item 25 ol 50 National Board of Medical Examiners^ Time Remaining:
9 Mark Comprehensive Basic Science Self-Assessment 1 hr 13 min 33 sec

25. A 23 -year- old woman is admitted to the hospital after taking an overdose of 16 acetaminophen tablets . She ingested the acetaminophen after she broke up with her boyfriend of 2 months. She has been admitted to the hospital several times during the past 5
years for treatment of suicide attempts and other self-harm attempts . Her history includes childhood sexual abuse and she has difficulty holding a job. She tells the physician that she has received substandard care from thiis hospital in the past. She says
; ;

"But now that you ' re my physician ! know I 'm going to get better. I can tell because we have a connection . ' Her affect is bright and cheerful. This patient most likely has which of the following types of personality disorders?
1

A ) Antisocial
0 B ) Borderline
O G ) Dependent
3 0 ) Marcissistic
(J E ) Schizoid

Previous *
Next Lab Values Calculator
m
Review
O
Help
m
Pause
Exam Section 3: Item 26 ol 50 National Board of Medical Examiners^ Time Remaining:
9 Mark Comprehensive Basic Science Self-Assessment 1 hr 13 min 30 sec

26. A 13 -year- old boy is scheduled to receive chemotherapy for a leukemia that has the histologic features of malignant lymphocytes . This neoplasm is further typed for cell surface and intracellular markers specific for lymphocyte subsets. The neoplastic cells do
not express the following markers : CD 4 . CDS : surface IgM, surface IgG , cytoplasmic IgM and p-heavy chain cytoplasmic lgG and y - heavy chain. The leukemic cells express class I MHC molecules and show rearrangement of the T-lymphocyte receptor 13-
,
:

chain gene D and J segments . Which of the following is the normal counterpart of these malignant lymphocytes?

O A ) Activated cytolytic effector!lymphocytes in the circulation


3 B ) Mature IgM -secreting B lymphocytes in the lymph node
O C ) Mature immunoglobulin- secreting plasma cells in the lymph node
3 0 ) Pre - B lymphocyte progenitor of mature B Jymphocytes in the bone marrow
E } T- lymphocyte thymocytes focalized to the thymic cortex

O
Previous *
Next Lab Values Calculator
m
Review
O
Help
m
Pause
Exam Section 3: Item 27 of 50 National Board of Medical Examiners^ Time Remaining:
9 Mark Comprehensive Basic Science Self-Assessment 1 hr 13 min 27 sec

27. The tracing shows contractile responses of a skeletal muscle preparation to electrical stimulation at five different frequencies. The amount of calcium sequestration in sarcoplasmic reticulum is highest during which of the following labeled responses?

A B C D E
H I/
O
c
0

Stimulus
1-Hz 3-Hz 6-Hz 9-Hz 12-Hz frequency

Time
O A)
O B)
O C)
O PJ
O E)

Previous Next Lab Values Calculator


m
Review
O
Help
m
Pause
Exam Section 3: Item 28 ol 50 National Board of Medical Examiners^ Time Remaining:
® Mark Comprehensive Basic Science Self-Assessment 1 hr 13 min 24 sec

28. A 33 -year- old man comes to the physician because of intermittent numbness and pain of his right arm during the past 6 months. His blood pressure is 120/80 mm Hg in both upper extremities: the pulse examination shows no abnormalities. A chest x-ray
shows a right complete cervical rib . A CT scan of the neck and chest shows compression of the brachial plexus . Which of the following vessels is at greatest risk for injury during operative removal of the right cervical rib ?

. A ) Brachiocephalic artery
B ) Right common carotid artery
. ) C ) Right internal carotid artery
_ D ) Right subclavian artery

Previous *
Next Lab Values Calculator
m
Review
O
Help
m
Pause
Exam Section 3: Item 29 of 50 National Board of Medical Examiners^ Time Remaining:
® Mark Comprehensive Basic Science Self-Assessment 1 hr 13 min 23 sec

29. A 24 -year- old man who is comatose is admitted to the hospital because of a drug overdose and pneumonia . On admission he is intubated and mechanically ventilated . While receiving a tidal volume of 500 ml_ . the patient ' s end - expiratory pressure is +5 cm
H 20 and his end-inspiratory airway pressure is + 25 cm H 20 . An esophageal balloon is inserted , and an end-inspiratory pleuraf pressure of + 20 cm Hp is measured . Which of the following best represents this patient's respiratory system compliance ?
:

A ) 0.01 cm Hp/mL
B ) 0.04 cm Hp /mL
o C ) 0.05 cm Hp/mL
o D ) 20 mLfcm HP
. E ) 25 niL/cm Hp
F ) 100 fflL/ cm Hp

Previous *
Next Lab Values Calculator
m
Review
O
Help
m
Pause
Exam Section 3: Item 30 ol 50 National Board of Medical Examiners^ Time Remaining:
® Mark Comprehensive Basic Science Self-Assessment 1 hr 13 min 20 sec

30 . A 5- month-old girl is brought to the physician because of a 3-day history of " floppiness" when she sits. She also has had difficulty feeding and irritability since the age of 1 month . She is at the 3rd percentile for length : 50th percentile for weight, and
3rd percentile for head circumference . Her temperature is 36°C (9G . BT ) , pulse is 10G/minp respirations are 29/ min and blood pressure is SO/ 50 mm Hg. Neurologic examination shows bilateral horizontal nystagmus decreased extensor tone: hyperreflexia .
;

and ankle clonus . Babinski sign is present bilaterally. Respiratory chain enzyme studies on cultured fibroblasts show decreased complex II activity. The most likely cause of these findings is a defect in which of the following processes?

O A ) Coupling of electron transport to proton gradient formation


3 B ) Coupling of proton gradient to ATP synthesis
o C) Oxidation of succinate to fumarate
3 0 ) Reduction of pyruvate to lactate
J E } Transfer of electrons from NADH to coenzyme Q
' J F ) Transfer of electrons from ubiquinol to cytochrome c

o
Previous Next Lab Values Calculator
9
Review
0
Help
0
Pause
Exam Section 3: Item 31 ol 50 National Board of Medical Examiners^ Time Remaining:
9 Mark Comprehensive Basic Science Self-Assessment 1 hr 13 min 17 sec

31. A 2 S -year- old woman who works as a glassblower has recently sustained burns on her hands taut feels little pain. Neurologic examination shows decreased pain sensation in taoth upper extremities and weakness and atrophy of the intrinsic muscles of both
hands. Which of the following best explains these findings?

. A ) Cystic fesion involving midline structures and both ventral horns in the cervical enlargement
8 ) Demyelination of the left corticospinal tract at all levels of the cord
O G ) Lesion of the left anterolateral quadrant at G-2
3 D ) Lass of myelin in the dorsal column white matter
'

. E ) Loss of myelin in the lateral column white matter and loss of ventral horn motoneurons

Previous *
Next Lab Values Calculator
m
Review
O
Help
m
Pause
Exam Section 3: Item 32 ol 50 National Board of Medical Examiners^ Time Remaining:
9 Mark Comprehensive Basic Science Self-Assessment 1 hr 13 min 15 sec

32 A 67-year- old woman has had decreased pain and temperature sensation in the left lower extremity for 1 week . The most iikely cause is damage to which of the following labeled sites in the cross section of the spinal cord?

Left Right

O A)
O 8)
O C)
O D)
O E)
O F)
O G)
O H)
OD
O J)

& C
Previous *
Next Lab Values Calculator Review
*
Help Pause
Exam Section 3: Item 33 of 50 National Board of Medical Examiners^ Time Remaining:
9 Mark Comprehensive Basic Science Self-Assessment 1 hr 13 min 10 sec

33. A 78 - year- old man is brought to the emergency department after falling from a ladder. On arrival, he is lucid . Physical examination shows a severe scalp contusion and laceration . Neurologic examination shows no
abnormalities. On the second day he is stuporous and has a headache . Examination shows slight weakness , hyperreflexis and an extensor plantar reflex , all on the right. A CT scan of the head is shown . The most likely
;

cause of these findings is trauma to which of the following vascular structures ?

A ) Artery in the subarachnoid space


Q S ) Dural bridging veins
Q C ) Middle cerebral artery
O 0 ) Middle meningeal artery
E ) Superior sagittal sinus

Previous *
Next Lab Values Calculator
m
Review
O
Help
m
Pause
Exam Section 3: Item 34 ol 50 National Board of Medical Examiners^ Time Remaining :
® Mark Comprehensive Basic Science Self-Assessment 1 hr 13 min 7 sec

34. A heafthy 24 -year- old woman participates in a study to determine the role of ghrelin in appetite . She is given free access to food during the study. The graph shows plasma concentrations of ghrelin in the woman during the study period . Which of the following
labeled points on the curve most likely represents the consumption of a meal?

c
o
tc

<D
c
o
£3

<U
OJ
CO
E/
< )
CO
CL

Ti me
O A)
O B)
O C)
O nj
O E)

O
Previous Next Lab Values Calculator
m
Review
O
Help
m
Pause
Exam Section 3: Item 35 ol 50 National Board of Medical Examiners^ Time Remaining :
® Mark Comprehensive Basic Science Self-Assessment 1 hr 13 min 4 sec

35. A 2 - month-old girl is brought to the emergency department by her mother because of difficulty breathing for S hours . Physical examination shows cyanosis . A loud systolic murmur is heard . Laboratory studies show hypocalcemia and lymphopenia.
Echocardiography shows tetralogy of Fallot . The diagnosis of DiGeorge syndrome is made , and a karyotype is ordered . Chromosomal analysis by high-resolution G -banding is reported as 46 ,XX . Which of the following genetic mechanisms is the most likely
cause of this patients anomalies ?

A ) Deletion
3 B ) Nondisjunction
o C ) Paracentric inversion
3 0 ) Pericentric inversion
J E } Reciprocal translocation
O F ) Robertsonian translocation
G ) Trinucleotide repeat
' J H ) Uniparental disomy

O
Previous Next Lab Values Calculator
m
Review
O
Help
m
Pause
Exam Section 3: Item 36 ol 50 National Board of Medical Examiners^ Time Remaining:
® Mark Comprehensive Basic Science Self-Assessment 1 hr 12 min 56 sec

36. A 6 S -year- old woman has had progressive shortness of breath with exertion since she started treatment with eyedrops for glaucoma 2 weeks ago. Pulmonary function tests show a decreased forced expiratory volume in 1 sec (FEV The lungs are dear to
auscultation. The eyedrops are most likely from which of the following classes of drugs? ^ .

. A ) o - Adrenergicagonist
6 ) p - Adrenergic agonist
O C) p-Adrenergic blocker
3 D ) Carbonic anhydrase inhibitor
. E ) Cholinesterase inhibitor
F ) Prostaglandin analog

o
Previous *
Next Lab Values Calculator
9
Review
0
Help
0
Pause
Exam Section 3: Item 37 of 50 National Board of Medical Examiners^ Time Remaining:
® Mark Comprehensive Basic Science Self-Assessment 1 hr 12 min 54 sec

37. A physician is sad because he has to inform a patient of recent test results that indicate progression of carcinoma to the terminal phase. When the patient sees the physician's face , he begins to cry and says "It ' s bad news , isn' t it?" Which of the following
;

responses by the physician is most appropriate?

A ) "How have you been since the last time I saw you?'
1

8 ) "Let' s talk about the positive aspects first /'


. C ) "Look on the bright side of things. "
D ) "Tell me how you are feeling. "
. E ) "There are other people who have it a lot worse than you."
O F ) "Yes, it is.11

G ) "You've had several years better off than many others with this disease. "

Previous *
Next Lab Values Calculator
m
Review
O
Help
m
Pause
Exam Section 3: Item 33 ol 50 National Board of Medical Examiners^ Time Remaining:
® Mark Comprehensive Basic Science Self-Assessment 1 hr 12 min 51 sec

38. An 18-year -old man comes to the physician because of nonpitting facial edema and difficulty breathing several hours after a dental procedure. He has a history of unexplained episodes of abdominal pain . His serum C 2 concentration is found to be markedly
decreased . Which of the foliowing best explains this patient' s illness ?

A ) Absent pyrin
8 ) Decreased C1 esterase inhibitors
0 C ) Increased urine porphyrins
D ) Penicillin allergy
. E } Superantigen production

O
Previous Next Lab Values Calculator
m
Review
O
Help
m
Pause
Exam Section 3: Item 39 of 50 National Board of Medical Examiners^ Time Remaining:
® Mark Comprehensive Basic Science Self-Assessment 1 hr 12 min 49 sec

39. A 67-year- old woman with acute myelogenous leukemia develops fever and chills 6 days after undergoing induction therapy. Her temperature is 38.3GC ( 100.9 QF) and blood pressure is 110 /60 mm Hg . Physical examination shows scattered petechiae over
;

the upper and lower extremities and trunk . Laboratory studies show a leukocyte count of 100/mms (98 % lymphocytes and 2% monocytes) and platelet count of 4000 / mmBlood cultures grow Pseudomonas aeruginosa . A deficiency of which of the following is
the most likely predisposing factor for development of bacteremia in this patient?

A ) Complement
0 B ) Immunoglobulin
0 G ) Lymphocytes
0 0 ) Macrophages
E ) Platelets
F ) Segmented neutrophils

o
Previous *
Next Lab Values Calculator
9
Review
0
Help
0
Pause
Exam Section 3: Item 40 ol 50 National Board of Medical Examiners^ Time Remaining:
® Mark Comprehensive Basic Science Self-Assessment 1 hr 12 min 46 sec

40 . A 54 -year- old woman with chronic kidney disease comes to the physician because of a 2 -week history of itchy skim increasingly severe nausea , and decreased urination. Current medications are hydrochlorothiazide and liisinopril. Her pulse is 80/min,
respirations are 14/min, and blood pressure is 160/90 mm Hg. Physical examination shows no abnormalities except for excoriations from itching . Laboratory studies show:
Hemoglobin 9.3 g/ dL
Hematocrit 30.1%
Mean corpuscular volume 89 pm 3
Leukocyte count 5600/mm 3
Platelet count 287,000/mm 3

Which of the following is the most likely cause of the findings in this patient?

A ) Hypoalbuminemia
B ) Hypothyroidism
C ) Uremia
D ) Vitamin C deficiency
'_) E ) Vitamin K deficiency

o
Previous *
Next Lab Values Calculator
m
Review
O
Help
m
Pause
Exam Section 3: Item 41 ol 50 National Board of Medical Examiners^ Time Remaining:
® Mark Comprehensive Basic Science Self-Assessment 1 hr 12 min 44 sec

41 . An 8 -year - old girl is brought to the physician for a well -child examination. Her mother says that she has been well except for an occasional cold . Her immunizations are up to date. She is at the 50th percentile for height and 60th percentile for weight. Physical
examination shows breast bud development and a few pubic hairs . The mother asks whether her daughter ' s development is normal. Which of the following is the most appropriate initial response by the physician?

. A ) '1 am concerned that your daughter may have precocious puberty. ' 1

8 ) " I think that your daughter is likely to begin menstruating in the next few months / '
0 C) '
! would like to order laboratory studies to ensure that your daughter ' s development is normal. '
1

' !
j D ) lYour daughter ' s breast development is somewhat less than might be expected for her age .”
E } 'Tour daughter ' s development is normal
.
^

Previous *
Next Lab Values Calculator
m
Review
O
Help
m
Pause
Exam Section 3: Item 42 ol 50 National Board of Medical Examiners^ Time Remaining:
® Mark Comprehensive Basic Science Self-Assessment 1 hr 12 min 41 sec

42 An 11 - year -old boy has had persistent pain in his right knee since he twisted it in a soccer game 3 weeks ago . Physical examination shows a tender area in the distal femur but no perceptible mass. An x-ray of the femur shows an osteolytic mass that has
eroded through the cortex and elevated the periosteum: the mass is surrounded by reactive bone in some areas. Biopsy of the mass shows atypical cells with hyperchromatic pleomorphic nuclei The cells are surrounded by an eosinophilic matrix : some of
which is calcified . This tumor is most likely to spread first to which of the following areas?

A ) Brain
3 B ) Cervical lymph nodes
G ) Liver
C 0 ) Lung
J E ) Vertebrae

Previous *
Next Lab Values
I*
Calculator
m
Review
O
Help
m
Pause
Exam Section 3: Item 43 of 50 National Board of Medical Examiners^ Time Remaining:
® Mark Comprehensive Basic Science Self-Assessment 1 hr 12 min 38 sec

43. A 52 -year- old man with lung cancer comes to the physician for a follow-up examination . Smoking cessation has been recommended, but the patient says that he is unable to quit because he experiences a pleasurable sensation as he inhales the smoke. He
has smoked 2 packs of cigarettes daily for 30 years . Which of the following mechanisms best explains how nicotine produces this sensation leading to addiction in this patient?

. A ) Activation of opioid receptors in the midbrain


8 ) increased release of dopamine in the nucleus accumbens
J C ) Inhibition of y-aminobutyric acid ( GABA ) release in the hypothalamus
3 D ) Inhibition of glutamate receptors in the amygdala
'

. E ) Potentiation of GABA receptors in the cerebral cortex

Previous *
Next Lab Values Calculator
m
Review
O
Help
m
Pause
Exam Section 3: Item 44 ol 50 National Board of Medical Examiners^ Time Remaining:
® Mark Comprehensive Basic Science Self-Assessment 1 hr 12 min 36 sec

44. A 65 - year- old man dies of renal failure . Urinalysis shows:


Blood negative
Protein negative
Glucose negative
Bence Jones protein positive

At autopsy: the kidneys are found to be enlarged . A histologic section of renal tissue is shown in the photomicrograph . Which of the following is the most likely diagnosis?

A ) Acute tubular necrosis


B ) Amyloidosis
O C jj Lipoid nephrosis
D ) Membranous glomerulonephritis

^ E ) Pyelonephritis

0
Previous
0
Next Lab Values Calculator
#
Review
0
Help
0
Pause
Exam Section 3: Item 45 ol 50 National Board of Medical Examiners^ Time Remaining:
® Mark Comprehensive Basic Science Self-Assessment 1 hr 12 min 33 sec

45. A 5 G -year- old man comes to the physician because of a 3- month history of a mild nonproductive cough . He has a 20 -year history of alcoholism . Physical examination shows no abnormalities . A PPD skin test result is positive. A chest x-ray and a Gram stain
,

of induced sputum show no abnormalities. Treatment with isoniazid and a daily multivitamin is begun. The patient is compliant with the isoniazid therapy but not with the multivitamin supplementation . Two months later he develops tingling and numbness of
both feet . Deficiency of which of the following is the most likely cause of the foot symptoms in this patient?

A ) Folic acid
0 B ) Niacin
0 G ) Vitamin B 2 (riboflavin)

0 D ) Vitamin B 6 (pyridoxine )
J E } Vitamin Bl2 ( cobalamin)

O
Previous *
Next Lab Values Calculator
m
Review
O
Help
m
Pause
Exam Section 3: Item 46 ol 50 National Board of Medical Examiners^ Time Remaining:
® Mark Comprehensive Basic Science Self-Assessment 1 hr 12 min 30 sec

46. Humoral antibody produced against many viruses following natural infection mediates protection against reinfection with the same strain primarily by which of the following mechanisms?

3 A ) Activation of macrophages
J S ) Blocking of viral attachment to cell receptors
( 3 G ) Inhibition of viral nucleic acid replication
J D ) Kilting of intracellular virus
'J E ) Prevention of release of virions from infected cells

O
Previous Next Lab Values Calculator
m
Review
O
Help
m
Pause
Exam Section 3: Item 47 ol 50 National Board of Medical Examiners^ Time Remaining:
® Mark Comprehensive Basic Science Self-Assessment 1 hr 12 min 28 sec

47. An investigator is studying a T- iymphocyte clone that recognizes a peptide from the hemaggfutinin glycoprotein of influenza vims bound by class II human leukocyte antigen . This clone most likely recognizes a peptide that was combined with a human
leukocyte antigen in which of the following cellular compartments ?

. A ) Cytosol
8 ) Endoplasmic reticulum
. C ) Endosomes
D ) Mitochondria
E ) Nucleus

Previous *
Next Lab Values Calculator
m
Review
O
Help
m
Pause
Exam Section 3: Item 43 ol 50 National Board of Medical Examiners^ Time Remaining:
® Mark Comprehensive Basic Science Self-Assessment 1 hr 12 min 25 sec

48. A 45 -year- old man is brought to the physician because of a 4-week history of mild confusion , memory difficulties , generalized muscle aching , and intermittent tingling of his lips and fingers . He says he has been sleeping more than usual , and that he feels
’down in the dumps ." Vital signs are normal . Physical examination shows no abnormalities except for dry: scaly skin and dry hair. Tapping with a reflex hammer over the left side of the face just anterior to the ear causes left facial muscle contractions. Which of
the following is the most likely diagnosis ?

A ) Addison disease
3 B ) Gushing disease
o C ) Hyperparathyroidism
3 0 ) Hyperthyroidism
E } Hypoparathyroidism
' F ) Hypothyroidism

o
Previous *
Next Lab Values Calculator
9
Review
0
Help
0
Pause
Exam Section 3: Item 49 of 50 National Board of Medical Examiners^ Time Remaining:
® Mark Comprehensive Basic Science Self-Assessment 1 hr 12 min 23 sec

49. A 38 - year- old man comes to the physician because of a 1-week history of a discharge from his penis and testicular pain . He is sexually active and uses condoms inconsistently. Physical examination shows a thick green
urethral discharge and exquisite tenderness of the right testis . The pain is relieved by elevation of the scrotum . A photomicrograph of a Gram stain of the discharge is shown . Which of the following is the most likely
causal organism?

A ) Chlamydia trachomatis
Q B ) Mycoplasma genitalium
o G ) Neisseria gortorrhoeae
O D ) Trichomonas vaginatis
E ) Ureaplasma urealyticum

Previous *
Next Lab Values Calculator
m
Review
O
Help
m
Pause
Exam Section 3: Item 50 ol 50 National Board of Medical Examiners^ Time Remaining:
® Mark Comprehensive Basic Science Self-Assessment 1 hr 12 min 21 sec

50 . One thousand office workers are surveyed to determine whether or not they use computers and whether or not they have symptoms of carpal tunnel syndrome. Which of the following best describes this study?

3 A ) Case- control study


3 S ) Case series
O G ) Cohort study
D ) Cross-sectional survey
3 E ) Randomized clinical trial

Previous *
Next Lab Values Calculator
m
Review
O
Help
m
Pause
Exam Section 4: Item 1 of 50 National Board of Medical Examiners^ Time Remaining:
9 Mark Comprehensive Basic Science Self-Assessment 1 hr 14 min 58 sec

1 . A male newborn is delivered vaginally at 39 weeks ' gestation to a 24- year -old primigravid woman. Tbe pregnancy was complicated by polyhydramnios. Soon after birth : the newborn has frothing at the mouth , cyanosis , and respiratory distress. An unsuccessful
attempt is made to pass a nasogastric tube . An x-ray shows an air-distended stomach. Which of the following is the most likely cause of these findings in this patient?

. A ) Abnormality of aortic arch resulting in vascular ring


8 ) Deviation of the tracheoesophageal septum
. C ) Failure of re-formation of duodenal lumen
!
'

D ) Hypertrophy of pyloric musde


. E ) Incomplete formation of the tracheal cartilage rings

Next Lab Values Calculator


9
Review
0
Help
0
Pause
Exam Section 4: Item 2 of 50 National Board of Medical Examiners^ Time Remaining:
9 Mark Comprehensive Basic Science Self-Assessment 1 hr 14 min 55 sec

2 . A 22 - year- old man is brought to the emergency department 30 minutes after a friend found him unconscious on the floor A drug overdose is suspected . His pulse is 120/minr respirations are 4/min with apparent low tidal volumes and blood pressure is
:

120/100 mm Hg. Physical examination shows mild cyanosis. He is intubated and mechanically ventilated with positive pressure ventilation ( rate = 12/min; VT = 7 mL/ kg) using 10 cm H 20 positive end -expiratory pressure (PEEP) . Which of the following sets of
>
findings best describes the effects of the mechanical ventilation with PEEP on this patient ' s alveolar (P J and intrapleural ( Pip pressures?

Peak inspiratory PA End- Tidal PA Peak Inspiratory Pjp End- Tidal Pip
O A) Positive positive positive positive
O B) Positive zero positive negative
O C) Zero zero zero negative
O D) Zero negative zero zero
O E) Negative positive positive positive
O F) Negative zero negative negative

Previous *
Next Lab Values Calculator
m
Review
O
Help
m
Pause
Exam Section 4: Item 3 of 50 National Board of Medical Examiners^ Time Remaining:
® Mark Comprehensive Basic Science Self-Assessment 1 hr 14 min 52 sec

3 . A 35 - year- old woman , gravida 1 , para 1 . comes to the physician for a follow-up examination 1 month after delivering a healthy male newborn. During her pregnancy, she was diagnosed with gestational diabetes. Her mother and sister also developed this
condition during pregnancy. Today, laboratory studies show a serum glucose concentration of 103 mg/dL . This patient most likely has decreased activity of which of the following enzymes?

. A ) D13 kinase
8 ) Glucokinase
O C ) Mevalonate kinase
3 D ) Phosphofructokinase
. . E } Pyruvate kinase

Previous *
Next Lab Values Calculator
m
Review
O
Help
m
Pause
Exam Section 4: Item 4 of 50 National Board of Medical Examiners^ Time Remaining:
9 Mark Comprehensive Basic Science Self-Assessment 1 hr 14 min 50 sec

4 . An 18-month- old boy is brought to the physician by his parents because of eye pain for 3 days. He has a history of multiple infections since the age of 3 weeks inctuding upper respiratory tract infections staphylococcal osteomyelitis and two episodes of
, , ,

pneumonia ( one due to Staphylococcus aureus and the other due to Aspergillus fumigatus ). Ophthalmologic examination shows erythema of the conjunctiva of the left eye with a small amount of purulent discharge . Physical examination shows multiple
ulcerated lesions with erythematous borders over the face , neck and back : cervical and axillary lymphadenopathy ; and hepatosplenomegafy. Serum IgE . !gG : and IgM concentrations are within the reference ranges. Culture of the conjunctival secretions grows
7

Staphylococcus epidermidis . and culture of several skin lesions grows Staphylococcus aureus . This patient most likely has which of the following immunodeficiency diseases ?

A ) Chediak-Higashi syndrome
O S ) Chronic granulomatous disease
0 C ) DiGeorge syndrome
) 0 ) Perinatal HIV infection
E ) Wiskott-Aldrich syndrome

O
Previous Next Lab Values Calculator
m
Review
O
Help
m
Pause
Exam Section 4: Item 5 of 50 National Board of Medical Examiners^ Time Remaining:
9 Mark Comprehensive Basic Science Self-Assessment 1 hr 14 min 47 sec

5 . A 10- year- old boy is brought to the physician by his parents because of increased urinary frequency and progressive fatigue during the past 3 weeks. Physical examination shows no abnormalities. His fasting serum glucose concentration is 350 mg/dL
Urinalysis shows 4+ glucose. Which of the following mechanisms in the proximal tubule is the most likely cause of the glycosuria in this patient?

. A ) Decreased sodium diffusion across the apical membrane


8 ) inhibition of the sodium,'' glucose cotransporter -1 ( SGLT- 1) carrier protein
O G ) Saturation of the gfucose transporter protein-1 (GLUT-1) carrier protein
3 D ) Saturation of glucose transporter protein-2 (GLUT-2) carrier protein
'

. E ) Saturation of the sodium / glucose cotransporter-1 ( SGLT- 1) carrier protein

O
Previous *
Next Lab Values Calculator
m
Review
O
Help
m
Pause
Exam Section 4: Item 6 of 50 National Board of Medical Examiners^ Time Remaining:
9 Mark Comprehensive Basic Science Self-Assessment 1 hr 14 min 45 sec

6 . A 5 -year- old boy has severe cholestatic liver disease and poor bile secretion . This patient is also likely to have a deficiency of vitamin E because its absorption requires which of the following?

0 A ) Binding to cholesterol
J B } Conversion to an active form by the liver
0 G ) Formation of micelles
0 D ) Intrinsic factor
E ) Trypsin

O
Previous Next Lab Values Calculator
m
Review
O
Help
m
Pause
Exam Section 4: Item 7 of 50 National Board of Medical Examiners^ Time Remaining:
® Mark Comprehensive Basic Science Self-Assessment 1 hr 14 min 40 sec

7 . A 23 -year- old woman comes to the physician because of a 1- week history of fatigue muscle pain and generalized weakness . She is unable to climb stairs because of the symptoms. She also has a 1-month history of a rash on her face: elbows, anti knuckles.
; ;

Her temperature is 38.3PQ ( 100.9 T }: pulse is 100/min, respirations are 20/min and blood pressure is 115 /65 mm Hg. Physical examination shows the findings in the photographs There is also proximal muscle weakness of the lower extremities bilaterally.
Which of the following is the most likely diagnosis?

o A ) Dermatomyositis
8 } Guillain Barre syndrome
-

o C ) Nephrotic syndrome
D ) Psoriasis
Q E ) Systemic lupus erythematosus

®
Previous
0
Next Lab Values Calculator
0
Review
0
Help
0
Pause
Exam Section 4: Item B of 50 National Board of Medical Examiners^ Time Remaining:
® Mark Comprehensive Basic Science Self-Assessment 1 hr 14 min 32 sec

8 . A 58 - year- old woman is brought to the emergency department 6 hours after the sudden onset of left- sided facial, arm and leg weakness. She has hypertension treated with a diuretic . She reports the sensation of a fluttering heart for 1 week. Her pulse is
125/min and irregular and blood pressure is 135 /80 mm Hg. Physical examination shows a left-sided facial droop. There is flaccid weakness of the left upper and lower extremities. An ECG shows a rapid : irregular ventricular rate with no discernable P waves.
Which of the following therapies would best improve function of the weakened muscles over the next 3 hours?

A ) Acetylcholine agonist
0 B ) Acetylcholine antagonist
0 G ) Antifibrinolytic
0 0 ) Fibrinolytic
J E } Procoagulant

O
Previous Next Lab Values Calculator
m
Review
O
Help
m
Pause
Exam Section 4: Item 9 of 50 National Board of Medical Examiners^ Time Remaining:
® Mark Comprehensive Basic Science Self-Assessment 1 hr 14 min 29 sec

9 . An investigator is planning to create gene therapy for Leigh syndrome, which is caused by an A ^ G mutation in the mitochondrial tRNALeu gene. Which of the following is the most likely reason why mitochondria encode their own tRNA?

3 A ) Mitochondria cannot import proteins or RNA

^ S ) Mitochondria produce large amounts of reactive oxygen species


( 3 G ) Mitochondria use a non- standard genetic code
J D ) The unusually high mitochondrial pH denatures nuclear - encoded tRNA
'J E ) The unusually low mitochondrial pH hydrolyzes nuclear - encoded tRNA

Previous *
Next Lab Values Calculator
m
Review
O
Help
m
Pause
Exam Section 4: Item 10 of 50 National Board of Medical Examiners^ Time Remaining:
® Mark Comprehensive Basic Science Self-Assessment 1 hr 14 min 26 sec

S i

10 . A 47- year- old man has microscopic blood on urinalysis . A CT scan of the abdomen shows a 5- cm mass in the left kidney A photomicrograph of tissue obtained on renal biopsy is shown. Which of the following is the fli

most likely diagnosis ?


N
'
V
C A ) Angiomyolipoma
B ) Nephroblastoma
. C ) Oncocytoma
O D ) Renal cell carcinoma
E ) Transitional cell carcinoma

O & C
Previous Next Lab Values Calculator Review
*
Help Pause
Exam Section 4: Item 11 of 50 National Board of Medical Examiners^ Time Remaining:
® Mark Comprehensive Basic Science Self-Assessment 1 hr 14 min 24 sec

11 . A 27 -year- old man comes to the physician because of fever and cough for 1 month . An x- ray of the chest shows diffuse infiltrates. Tiny intracellular yeast forms are seen on direct smears and tuberculate macroconidia are seen after culture at room
;

temperature . Which of the following exposures is most consistent with this patient's illness?

. A ) Bird droppings
B ) Desert sandstorm
O C ) Insect bite
_) D ) Moldy hay
'

. E } Rose thorn puncture

O
Previous *
Next Lab Values Calculator
m
Review
O
Help
m
Pause
Exam Section 4: Item 12 of 50 National Board of Medical Examiners^ Time Remaining:
® Mark Comprehensive Basic Science Self-Assessment 1 hr 14 min 22 sec

12 A 63 -year- old man with type 2 diabetes mellitus hypertension, and hyperlipidemia comes to die physician because of pain with urination blood in his urine and fatigue during the past week . He has a 6-month history of increased urinary frequency with small
; ;

volume and nighttime urination . He has had two episodes of acute cystitis during the past 8 months. His temperature is 38.3°C ( 100.9 CF ) : pulse is 92/min , respirations are 20/mm, and blood pressure is 108 /72 mm Hg . Physical examination shows suprapubic
tenderness and an enlarged soft prostate. Ultrasonography shows a bladder diverticulum and enlarged kidneys with dilation of renal petves and calyces. A urine culture grows Escherichia coli . Which of the following is the most likely underlying cause of this
,

patient' s condition?

A ) Benign prostatic hyperplasia


. B ) Hypertension
0 C ) Nephrolithiasis
, 0 ) Type 2 diabetes mellrtus
E ) Xanthogranulomatous cystitis

Previous *
Next Lab Values Calculator
m
Review
O
Help
m
Pause
Exam Section 4: Item 13 of 50 National Board of Medical Examiners^ Time Remaining:
9 Mark Comprehensive Basic Science Self-Assessment 1 hr 14 min 19 sec

13. As part of an annual physical examination a healthy 5G-year-aEd man has laboratory studies done 3 hours after an overnight fast . His serum glucose concentration is 75 mg/dL Activation of which of the following receptors in liver cells most likely maintained
the serum glucose concentration in this patient?

A ) Cytokine
8 ) G protein-coupled
. . C ) Nuclear
D ) Serine kinase
. E ) Tyrosine kinase

Previous *
Next Lab Values Calculator
m
Review
O
Help
m
Pause
Exam Section 4: Item 14 of 50 National Board of Medical Examiners^ Time Remaining:
9 Mark Comprehensive Basic Science Self-Assessment 1 hr 14 min 17 sec

-
14 . A 73-year- old man with stage IV colon cancer begins chemotherapy with 5 fluorouracil - irinotecan. and leucovorin. This patient is at greatest risk for developing signs of drug toxicity in which of the following tissues?

3 A ) Bone marrow, gut mucosa and hair follicles


,

Q^ B } Liver, kidney, and endocrine organs


G ) Nerves, bone : and heart muscle
J D ) Skeletal muscle heart and bone
:

'J E ) Trachea , bronchial epitheiiuni and liver

Previous Next Lab Values Calculator


m
Review
O
Help
m
Pause
Exam Section 4: Item 15 of 50 National Board of Medical Examiners^ Time Remaining:
9 Mark Comprehensive Basic Science Self-Assessment 1 hr 14 min 14 sec

15. An investigator is conducting a study of the lipopolysaccharide synthesis pathway. During the study a polymerase chain reaction test is developed that detects a gene involved in lipid A biosynthesis. Which of the following organisms is most likely to yield a
,

positive result using this polymerase chain reaction test?

. A } Candida albicans
8 ) Clostridium difficile
O C ) Mycoplasma pneumoniae
0 D ) Pasteuretia multocida
C E ) Staphylococcus aureus

Previous *
Next Lab Values Calculator
m
Review
O
Help
m
Pause
Exam Section 4: Item 16 of 50 National Board of Medical Examiners^ Time Remaining:
® Mark Comprehensive Basic Science Self-Assessment 1 hr 14 min 11 sec

16. A 25 -year- old woman comes to the physician because of general malaise and a facial rash for 1 week . She has a 10-year history of episodes of pleurisy and arthritic pain in peripheral joints . Physical examination shows an erythematous malar rash that does
not involve the nasolabial folds . Resuits of cardiolipin antibody anti -dsDNA , and anti-Sm antibody assays are positive. Which of the following hematologic abnormalities is most likely in this patient?
;

. A ) Hemolytic uremic syndrome


8 ) Macrocytic anemia
. C ) Multiple nucleated erythrocytes
! 0 ) Rouleaux formations
. E } Thrombocytopenia

O
Previous
0
Next Lab Values Calculator
0
Review
0
Help
0
Pause
Exam Section 4: Item 17 of 50 National Board of Medical Examiners^ Time Remaining :
® Mark Comprehensive Basic Science Self-Assessment 1 hr 14 min 7 sec

17. Which of the following characteristics of the glucocorticoid receptor is the most likely cause of the persistence of the pharmacologic effects of dexamethasone after the drug is eliminated from the body ?

3 A ) Catalyzes formation of stable tyrosine-phosphate bonds


i S ) Competes with p- arrestin for binding to adrenergic receptors
O C ) Couples to G proteins with low intrinsic GTPase activity
J D ) Depletes cellular stores of glutathione
'J E ) Induces proteins that remain functional after the drug is eliminated

Previous *
Next Lab Values Calculator
m
Review
O
Help
m
Pause
Exam Section 4: Item 13 of 50 National Board of Medical Examiners^ Time Remaining :
® Mark Comprehensive Basic Science Self-Assessment 1 hr 14 min 4 sec

18. A 23 -year- ofd woman comes to the physician for advice on howto lose weight . She teIJs the physician that she binges on high- carbohydrate foods two to three times weekly, usually forcing herself to vomit after a binge . She is 168 cm (5 ft 6 in) tall and weighs
63 kg ( 140 lb); BMl is 23 kg/m 2 Which of the following additional physical findings is most likely in this patient?

A ) Bradycardia
B ) Decreased vibratory sensation at the ankles
. C ) Parotid gland enlargement
!
'

D ) Sluggish deep tendon reflexes


. E } Sparse axillary and pubic hair

Previous *
Next Lab Values Calculator
m
Review
O
Help
m
Pause
Exam Section 4: Item 19 of 50 National Board of Medical Examiners^ Time Remaining :
® Mark Comprehensive Basic Science Self-Assessment 1 hr 14 min 0 sec

19. Strips of tracheal muscle contract after administration of carbachol then relax after administration of epinephrine. The graph shows muscle tension as a function of the log dose of epinephrine under
control conditions (solid curve ) and in the presence of equal concentrations of antagonists X ( dashed curve) and Y ( dotted curve). X and Y are most likely to be which of the following drugs?
cn

Drug X Drug Y =
£
O
CO

O A) Clonidine yohimbine £Z
OJ

O B) Metoprolol propranolol OJ
o
CO

O C) Phentoiamine prazosin = 3

**

O D> Prazosin phentoiamine


Log [ epinephrine]
O &) Propranolol metoprolol
O F) Yohimbine clonidine

O
Previous Next Lab Values Calculator
m
Review
O
Help
m
Pause
Exam Section 4: Item 20 ol 50 National Board of Medical Examiners^ Time Remaining:
9 Mark Comprehensive Basic Science Self-Assessment 1 hr 13 min 57 sec

20 . A 19-year- old woman comes to the physician 6 hours after the sudden onset of left-sided , cramping abdominal pain. She does not have nausea . She also has a 3 -week history of pain with sexual intercourse , and she is sexually active with three male
partners. Her last menses started 5 days ago. Current medication is an oral contraceptive. Her temperature is 38 CC ( 1D 0.4°F) . Physical examination shows a normal cervix with no discharge . Bimanual examination shows tenderness and fullness on the left
side compared with the right and cervical motion tenderness . A urine pregnancy test result is negative . Which of the following structures is most likely affected by this patient' s current condition?
,

A ) Appendix
O '
B ) Bladder
G ) Endometrium
0 0 ) Kidney
J E ) Peritoneum
F ) Uterine (fallopian) tube

Previous *
Next Lab Values Calculator
m
Review
O
Help
m
Pause
Exam Section 4: Item 21 ol 50 National Board of Medical Examiners^ Time Remaining:
9 Mark Comprehensive Basic Science Self-Assessment 1 hr 13 min 53 sec

21 . A 21-year- old woman with primary pulmonary hypertension begins treatment with bosentan . As a result blockade of which of the following is most likely to occur?
,

0 A ) Angiotensin II receptors
0 S ) Calcium channels
0 G ) Endothelin receptors
J D ) Production of phosphodiesterase 5
'J E ) Voltage- gated sodium channels

O
Previous Next Lab Values Calculator
m
Review
O
Help
m
Pause
Exam Section 4: Item 22 ol 50 National Board of Medical Examiners^ Time Remaining:
9 Mark Comprehensive Basic Science Self-Assessment 1 hr 13 min 51 sec

22 A GG-year- old woman comes to the physician because of a 1 -month history of bleeding from a lesion on her nose . She has no history of major medical illness and takes no medications . Physical examination shows a 1- cm lesion on the right naris. Microscopic
examination of a biopsy specimen of the mass shows neoplastic cells that exhibit dense pigment granules . Which of the following is the most likely diagnosis?

. A ) Actinic keratosis
8 ) Basal cell carcinoma
. C ) Melanoma
3 D ) Rhabdomyosarcoma
. E } Squamous cell carcinoma

O
Previous *
Next Lab Values Calculator
m
Review
O
Help
m
Pause
Exam Section 4: Item 23 of 50 National Board of Medical Examiners^ Time Remaining:
® Mark Comprehensive Basic Science Self-Assessment 1 hr 13 min 4£ sec

23. A 13-year- old girl is brought to the physician because of a mass ifi the thoracic region that has been enlarging during the past month . She was placed in a foster home 1 year ago . She is at the 70th percentile for height and weight . Breast and pubic hair
development is Tanner stage 2. Physical examination shows a 3-mm mildly pigmented mass located in the midclavicular ine at the 10th rib . The mass has a smooth surface uniform outer border and central papule. There are no other pigmented lesions.
; ,

Which of the following is the most likely diagnosis?

A ) Accessory nipple
3 B ) Benign pigmented nevus
G ) Melanoma
3 0 ) Neurofibroma
J E } Seborrheic keratosis

O 9 0 0
Previous *
Next Lab Values Calculator Review Help Pause
Exam Section 4: Item 24 ol 50 National Board of Medical Examiners^ Time Remaining:
9 Mark Comprehensive Basic Science Self-Assessment 1 hr 13 min 45 sec

24. A 52 -year- old woman comes to the emergency department because of a 2-week history of progressive shortness of breath and fatigue. Her pulse is 102/mrn, respirations are 22/ min , and blood pressure is 100/Bu mm Hg . Physical examination shows muffled
breath sounds. Echocardiography shows a large pericardial effusion . Pericardiocentesis yields dowdy, serosanguineous fluid . Analysis of the fluid shows an increased protein concentration numerous RBCs . and a small number ofWBCs . indicative of
malignancy. Metastasis from which of the following sites is the most likely cause of the findings in this patient?

A ) Bladder
0 B ) Breast
o C ) Colon
0 0 ) Ovaries
J E ) Stomach

O
Previous Next Lab Values Calculator
m
Review
O
Help
m
Pause
Exam Section 4: Item 25 ol 50 National Board of Medical Examiners^ Time Remaining:
® Mark Comprehensive Basic Science Self-Assessment 1 hr 13 min 42 sec

25. A 5 -month -old boy is brought to the emergency department because of a 3- day history of fever and severe wet cough . His temperature is 40.5* 0 (104.9 aF ) and respirations are 65/min Crackles are
;

heard over all lung fields. A photomicrograph of a silver-stained specimen obtained via bronchoalveolar lavage is shown. In addition to a lack of expression of human leukocyte antigen - DR molecules
by lymphocytes , flow cytometry of a peripheral blood specimen will most likely show markedly decreased populations of which of the following cell types?

A ) CD 4+- T lymphocytes
S ) Dendritic cells
C ) Monocytes
O Q ) Natural killer cells
E ) Segmented neutrophils

O
Previous Next Lab Values Calculator
m
Review
O
Help
m
Pause
Exam Section 4: Item 26 ol 50 National Board of Medical Examiners^ Time Remaining:
9 Mark Comprehensive Basic Science Self-Assessment 1 hr 13 min 39 sec

26. A 29-year- old woman comes to the physician because of irregular menstrual periods since menarche at the age of 12 years. She is 1GO cm (5 ft 3 in) tail and weighs 86 kg (190 lb ) ; BMI is 34 kg/m 2 She is evaluated and the diagnosis of polycystic ovarian
,

syndrome is made. After explaining the diagnosis, the physician discusses behavioral changes including dietary modification and exercise as part of her treatment plan . Which of the following is most likely to result in patient adherence to this plan?
,

O A ) Ascertain the patient ' s educational level and provide appropriate publications
B ) Ask the patient to bring a family member or friend to the next appointment
. J C ) Inform the patient of the health consequences of not treating her condition
3 D ) Provide follow-up appointments to assess the patient's progress in attaining her goals
'

. E } Refer the patient to a support group

O 9 0 0
Previous *
Next Lab Values Calculator Review Help Pause
Exam Section 4: Item 27 of 50 National Board of Medical Examiners^ Time Remaining:
9 Mark Comprehensive Basic Science Self-Assessment 1 hr 13 min 37 sec

27. A study is conducted to determine if concomitant oral administration of ciprofloxacin and sucralfate results in a decreased plasma ciprofloxacin concentration. Twelve subjects are enrolled in the study ; six receive ciprofloxacin only and six receive both
,

treatments . The half- life is determined, and a 1-week washout period follows. Each subject then receives the other treatment and haff-life is measured again . Which of the following best describes the design of this study?

A ) Case- control
8 ) Case series
. C ) Crossover
3 D ) Historical cohort
. E ) Prospective cohort

Previous *
Next Lab Values Calculator
m
Review
O
Help
m
Pause
Exam Section 4: Item 28 ol 50 National Board of Medical Examiners^ Time Remaining:
9 Mark Comprehensive Basic Science Self-Assessment 1 hr 13 min 33 sec

28. A 32 -year- old woman is found to have panic disorder with agoraphobia . A drug is prescribed that activates benzodiazepine binding sites on the y- smmobutyric acid A (GABA J receptor. This drug is most likely which of the following?

A ) Alprazolam
S ) Buspirone
O C ) Flumazenil
3 D ) Hydroxyzine
3 E ) Ramefteon

O
Previous *
Next Lab Values Calculator
9
Review
0
Help
0
Pause
Exam Section 4: Item 29 of 50 National Board of Medical Examiners^ Time Remaining:
® Mark Comprehensive Basic Science Self-Assessment 1 hr 13 min 31 sec

29. A 1 - year- old African American girl is brought to the physician because of a slow growth rate . At birth she was at the 25th percentile for length and 30th percentile for weight . She has been exclusively breast-fed since birth. Her length and weight are now at the
5th percentile . Physical examination shows bowed legs swelling of the wrists and a bulging of the costochondral junctions . Her serum calcium concentration is within the reference range and her serum phosphorus concentration is decreased . Which of the
, , ,

following sets of serum findings is most likely in this patient?

Parathyroid Hormone 2 5 -Hydroxy vitamin D


O A) T T
O B) T I
O C) Normal I
O D) i normal
O E) ! T
O F) i 1

Previous *
Next Lab Values Calculator
m
Review
O
Help
m
Pause
Exam Section 4: Item 30 ol 50 National Board of Medical Examiners^ Time Remaining:
9 Mark Comprehensive Basic Science Self-Assessment 1 hr 13 min 27 sec

30 . A premenopausal 49-year -old woman asks her physician about her risk for osteoporosis. Her mother had osteoporosis and disabling bone fractures after menopause. The patient works as a gardener for a landscape service and plays tennis 3 times a week .
She does not smoke drinks 1 glass of wine a week, and takes no medications. Her weight is 55 kg ( 121 lb). Which of the following additional information is needed to evaluate her risk for osteoporosis ?
7

. A ) Age at first sexual intercourse


8 ) Dietary history
. J C ) Exposure to ionizing radiation
3 D ) Exposure to organophosphate
. E ) Family history of rheumatic diseases
F ) Socioeconomic history

Previous *
Next Lab Values Calculator
m
Review
O
Help
m
Pause
Exam Section 4: Item 31 of 50 National Board of Medical Examiners^ Time Remaining:
® Mark Comprehensive Basic Science Self-Assessment 1 hr 13 min 24 sec

31. A 5 - year- old boy is brought to the physician by his parents because of lethargy for 1 week. He has a history of occasional colds and ear infections. Physical examination shows mild pitting edema of the lower extremities . Auscultation of the chest and palpation
of the abdomen show no abnormalities. Laboratory studies show :
Hemoglobin 15 g /dL
Hematocrit 45%
Leukocyte count 7300/mms
Serum cholesterol 380 mg.' dL
Urine
Blood absent
Protein 3+

If a kidney biopsy specimen were obtained : light microscopic examination would most likely show which of the following?

A ) Crescents within the glomerular space


B ) Focal segmental glomerulosclerosis
O C ) Neutrophilic infiltration of the glomerular tuft
J D ) Thickened glomerular basement membranes
E ) No pathologic abnormality

O
Previous *
Next Lab Values Calculator
m
Review
O
Help
m
Pause
Exam Section 4: Item 32 ol 50 National Board of Medical Examiners^ Time Remaining:
® Mark Comprehensive Basic Science Self-Assessment 1 hr 13 min 21 sec

32 Feedback inhibition of the activity of the enzyme that catalyzes this reaction is critical in the control of the de novo synthesis of which of the following?
®-o— H
O A ) Complex carbohydrates H
B ) Folates
OH OH
Q C ) Pentose phosphates
Glutamine
D ) Purine nucleotides lutamate
SPFi
Q E ) Pyrimidine nucleotides
©- O- CHj ^O
NHa
H
H \H H

OH OH

Previous Next Lab Values Calculator


o
Review
o
Help
o
Pause
Exam Section 4: Item 33 of 50 National Board of Medical Examiners^ Time Remaining:
9 Mark Comprehensive Basic Science Self-Assessment 1 hr 13 min 18 sec

33. A 72 -year- old woman with coronary artery disease comes to the physician because of a 2 -month history of progressive angina symptoms with exertion. Five months ago , the patient underwent stent placement for significant stenoses of the proximal anterior
interventricular (left anterior descending) and right coronary arteries . She remained symptom-free for 3 months . Her pulse is 76/ min and regular and biood pressure is 135 /85 mm Hg . An EGG at rest shows no abnormalities : an exercise stress test shows ST-
segment changes in leads II , III and aVF. Which of the following is the most likely cause of this patient' s recurrent angina symptoms ?

A ) Aneurysm of the right coronary artery


3 B ) Dissection of the right coronary artery
o C ) Neointima formation in the right coronary stent
3 D ) Thrombosis of the right coronary stent
E } Vasospasm in the stented right coronary artery

Previous *
Next Lab Values Calculator
m
Review
O
Help
m
Pause
Exam Section 4: Item 34 ol 50 National Board of Medical Examiners^ Time Remaining:
9 Mark Comprehensive Basic Science Self-Assessment 1 hr 13 min 15 sec

34. A 55 -year- old man comes to the physician because of a 3- week history of night sweats weakness fatigue , loss of appetite, and lumps in his neck . He owns a printing business and works with paints and soEvents . Physical examination shows edematous
, ,
:

tender axillary and inguinal lymph nodes . His leukocyte count is 45.570/mm 3 (30% large pleomorphic blasts with Auer rods ). Occupational exposure to which of the following most likely contributed to the development of this patient' s disease ?
,

. A ) Benzene
B ) Cadmium
0 C ) Chlordane
0 D ) Naphthylamine
. E } Vinyl chloride

O
Previous Next Lab Values Calculator
m
Review
O
Help
m
Pause
Exam Section 4: Item 35 ol 50 National Board of Medical Examiners^ Time Remaining:
® Mark Comprehensive Basic Science Self-Assessment 1 hr 13 min 11 sec

35. A matched case-control study is conducted to assess the association between the frequent use of tanning beds during teenage years and the development of melanoma . A group of women with melanoma are matched with a group of cancer -free women of
the same age and ethnicity. An odds ratio (OR) of 1.4 is obtained . Which of the following 95% confidence intervals for the OR is the most precise statistically significant interval?

A ) -0.9 to 0.9
O 0 ) 0.9 to 1.7
O C ) 0.98 to 1.01
o D ) 1.01 to 1.73
. E ) 1.2 to 1.7

O
Previous *
Next Lab Values Calculator
m
Review
O
Help
m
Pause
Exam Section 4: Item 36 ol 50 National Board of Medical Examiners^ Time Remaining :
® Mark Comprehensive Basic Science Self-Assessment 1 hr 13 min 9 sec

36. A G - week-old male infant has a persistent discharge from a swollen umbilical stump. If the discharge contains intestinal fluid to which of the following portions of the gastrointestinal tract is the fistula most likely connected?
,

O A ) Cecum

^0 B ) Duodenum
G ) ileum
0 D ) Jejunum
E ) Sigmoid colon

Previous *
Next Lab Values Calculator
m
Review
O
Help
m
Pause
Exam Section 4: Item 37 of 50 National Board of Medical Examiners^ Time Remaining :
® Mark Comprehensive Basic Science Self-Assessment 1 hr 13 min 5 sec

37. A 32 -year- old man comes to the physician because of a 6- month history of progressive drooping of the eyelids and facial weakness. Muscle weakness is exacerbated by use and relieved by rest. If a neoplasm is the cause of these findings which of the
,

following is the most likely location in this patient?

A ) Femur
8 ) Liver
. C ) Lymph nodes
'

_ D ) Meninges
. . E ) Thymus

Previous *
Next Lab Values Calculator
m
Review
O
Help
m
Pause
Exam Section 4: Item 33 ol 50 National Board of Medical Examiners^ Time Remaining :
® Mark Comprehensive Basic Science Self-Assessment 1 hr 13 min 2 sec

38. When control lymphocytes are treated with corticosteroids , a majority of cells shrink in size and develop peripheral chromatin condensation ; cytoplasmic organelles are intact. DNA isolated from the control lymphocytes: electrophoresed on an agarose gel and
stained with ethidium bromide, shows a ladder of regularly spaced hands . When Eymphocytes from the same culture are transfected so that they overexpress Gene X . which encodes a normal protein product the cells continue to proliferate and do not
undergo any of the morphologic changes seen in the control cells. Gene X is most likely to encode which of the following?

A ) BCL2 protein
0 B ) ERBB2 protein
G ) p 53 protein
3 0 ) Platelet- derived growth factor
E ) Rb protein

Previous *
Next Lab Values Calculator
m
Review
O
Help
m
Pause
Exam Section 4: Item 39 of 50 National Board of Medical Examiners^ Time Remaining:
9 Mark Comprehensive Basic Science Self-Assessment 1 hr 12 min 59 sec

39. A 65 -year- old man comes to the physician for a follow-up examination. He has a 15 -year history of poorly controlled type 2 diabetes mellitus , resulting iin multiple peripheral neuropathies . Neurologic examination shows wasting of the interosseous muscles of
the left hand and inability to abduct the f ngers of this hand: plantar flexion of the right foot is absent The function of which of the following pairs of nerves is most likely impaired in this patient?

. A ) Median and common fibular (peroneal)


8 ) Median and tibial
. J C ) Radial and common fibular (peroneal)
D ) Radial and tibial
. E } Ulnar and common fibular (peroneal)
F ) Ulnar and tibial

Previous *
Next Lab Values Calculator
m
Review
O
Help
m
Pause
Exam Section 4: Item 40 ol 50 National Board of Medical Examiners^ Time Remaining:
® Mark Comprehensive Basic Science Self-Assessment 1 hr 12 min 56 sec

40 A 12 -year- old girl is brought to the physician because of a rash on her let! buttock for the past 2 days. The rash developed after the family returned from a 2 - week-long early summer vacation in Maine. Her vital signs are within normal limits. A photograph of
the lesion is shown . The likely cause of this patient ' s infection is taxonomicalfy and morphologically most similar to the infectious agent of which of the following conditions ?

A ) Bacillary angiomatosis
O B ) Chancroid
C ) Leptospirosis
J D } Lymphogranuloma venereum
. _ E ) Q fever

®
Previous
0
Next Lab Values Calculator
0
Review
0
Help
0
Pause
Exam Section 4: Item 41 ol 50 National Board of Medical Examiners^ Time Remaining:
9 Mark Comprehensive Basic Science Self-Assessment 1 hr 12 min 53 sec

41 . A single oral . 6-mg dose of a short-acting (3 agonist is studied to determine its pharmacokinetics and pharmacodynamics in young subjects with stable mild asthma . In this group , the Cma:/AUC following oral dosing varies by less than &%. However, in 30%
,
of subjects : the maximal FEV increase is 25^
,

% lower than that of the other subjects. Which of the following mechanisms best explains these observations in this subgroup ?

A ) Decreased catechol O- methyltransferase activity in bronchial smooth muscle


8 ) Decreased phosphodiesterase IV in bronchial smooth muscle
. J C ) Increased gastrointestinal P- glycoprotein
3 D ) Receptor polymorphism with reduced signal transduction
'

(_ y E ) Receptor up regulation

Previous *
Next Lab Values Calculator
m
Review
O
Help
m
Pause
Exam Section 4: Item 42 ol 50 National Board of Medical Examiners^ Time Remaining:
® Mark Comprehensive Basic Science Self-Assessment 1 hr 12 min 50 sec

42 A 19- year- old man comes to the emergency department because of increasingly severe shoulder and abdominal pain for 3 days. His temperature is 3EPG (102.2T ). Physical examination shows signs
of acute peritonitis. An abdominal x- ray is shown. Which of the following is the most likely cause of this patient's current condition?

A ) Ileus
Q B ) Intraperitoneal abscess
C ) Nephrolithiasis
O D ) Perforated viseus
E ) Retained foreign body

O
Previous Next Lab Values Calculator
m
Review
O
Help
m
Pause
Exam Section 4: Item 43 of 50 National Board of Medical Examiners^ Time Remaining:
9 Mark Comprehensive Basic Science Self-Assessment 1 hr 12 min 47 sec

43. A 2 S -year- old man is participating in a study of the effect of altitude and conditioning on combat readiness . He is a Special Forces sergeant in the army. While he exercises in an environmental chamber, the ambient Po 2 is decreased from 160 mm Hg to 60
mm Hg. The decrease in ambient Po ? will most likely cause an increase in which of the following in this patient?

. A ) Arterial diastolic pressure


8 ) Arterial pulse pressure
O G ) Arterial systolic pressure
3 D ) Mean arterial pressure
'

. E } Mean pulmonary artery pressure

Previous *
Next Lab Values Calculator
m
Review
O
Help
m
Pause
Exam Section 4: Item 44 ol 50 National Board of Medical Examiners^ Time Remaining:
® Mark Comprehensive Basic Science Self-Assessment 1 hr 12 min 44 sec

44. A 52 -year- old woman is admitted to the hospital because of chest pain and shortness of breath for 2 hours. Her pulse is 102/mia respirations are 35/min , and blood pressure is 110/65 mm Hg . Cardiac examination shows a laterally displaced apex, a systolic
thrill and a grade 4 /6 crescendo-decrescendo systolic ejection murmur. Cardiac catheterization shows a pulmonary capillary wedge pressure of 40 mm Hg ( N=5—16). Which of the following sets of changes in this patient' s left ventricle is most likely ?
,
:

Stroke Myocardial Oxygen Myocardial Oxygen Adenosine


Work Consumption Tension Conce ntration
O A) Increased increased increased decreased
O B) increased increased decreased increased
O C) Increased decreased increased increased
O D) increased decreased decreased increased
O E) Decreased increased increased increased
O F) Decreased increased increased decreased
O G) Decreased decreased increased increased
O H) Decreased decreased decreased decreased

O 9 0 0
Previous *
Next Lab Values Calculator Review Help Pause
Exam Section 4: Item 45 ol 50 National Board of Medical Examiners^ Time Remaining:
® Mark Comprehensive Basic Science Self-Assessment 1 hr 12 min 41 sec

45. A 65 -year- old woman with severe sensorineural hearing loss undergoes surgical placement of a cochlear impEant . This neural prosthesis converts sound energy to electrical signals which resuits in stimulation of which of the following structures ?
,

3 A ) Auditory nerve endings in the cochlea


J 8 } Cochlear nerve as it enters the pons
( J: G ) Inner hair cells of the cochlea
J D ) Olivocochlear efferent axons that innervate outer hair celfs
'J E ) Oval window of the cochlea

Previous *
Next Lab Values Calculator
m
Review
O
Help
m
Pause
Exam Section 4: Item 46 ol 50 National Board of Medical Examiners^ Time Remaining:
® Mark Comprehensive Basic Science Self-Assessment 1 hr 12 min 38 sec

46. A 4 S -year- old man begins furosemide therapy for pedal edema associated with biventricular failure and hypertension. Five days later, the edema is not fully resolved and his serum potassium concentration has decreased from 4.2 mEq/L to 3 mEq/ L . A drug
with which of the following actions should be added to this patient's medication regimen?

. A ) Blocks basolateral K + channels in the collecting duct


B ) Decreases the luminal permeability to Na + in the collecting duct
. J C ) Increases the delivery of Na + and K + to the collecting duct
3 D ) increases the negative charge of the luminal tubule fluid
'

. . E } Stimulates NaMOATPase in the collecting duct

Previous *
Next Lab Values Calculator
m
Review
O
Help
m
Pause
Exam Section 4: Item 47 of 50 National Board of Medical Examiners^ Time Remaining:
® Mark Comprehensive Basic Science Self-Assessment 1 hr 12 min 35 sec

47. A 70- year- old man comes to the physician because of a 4-month history of fatigue and cough productive of blood-tinged sputum. He has a history of hyperlipidemia . He had smoked 1 pack of cigarettes daily for
30 years until he quit 5 years ago . His temperature is 37.2 GC ( 993F) , pulse is 120/mirj, respirations are 20/ min : and blood pressure is 130/85 mm Hg . Diffuse crackles and wheezes are heard on inspiration . A chest
x- ray is shown . Serum studies show :
Na + 138 mEq/ L
K+ 5.0 mEq/L
Cl- 102 mEq/ L
HCG,- 24 mEq /L
Ca 2 + 15 mg/dL

In addition to intravenous hydration and induction of diuresis to treat his hypercalcemia, it is most appropriate for this patient to begin therapy with a drug that inhibits the activity of which of the Mowing cells?

A ) Chondroblasts

^ B ) Chondrocytes
C ) Osteoblasts
_ ! D ) Osteoclasts
E ) Parathyroid cells

Previous *
Next Lab Values Calculator
#
Review
&
Help
0
Pause
Exam Section 4: Item 43 ol 50 National Board of Medical Examiners^ Time Remaining:
® Mark Comprehensive Basic Science Self-Assessment 1 hr 12 min 32 sec

48. A 22 - year- old man comes to the physician because of a 2 -month history of foul-smelling watery diarrhea with significant flatulence: he also has had a 4.5-kg (10-lb) weight loss during this period . He recently returned
,

from a trip to rural Indonesia , where he did not always have access to clean water. He appears thin . Physical examination shows a soft mildly distended abdomen with active bowel sounds . A photomicrograph of a stool
specimen is shown . The most appropriate pharmacotherapy has which of the following mechanisms of action?

A ) Enhancement of cell membrane permeability to chloride ions


Q S ) Formation of destructive free radicals
C ) Inhibition of DNA polymerase
O 0 ) Inhibition of protein synthesis
E ) Prevention of microtubule assembly

& C
Previous *
Next Lab Values Calculator Review
*
Help Pause
Exam Section 4: Item 49 of 50 National Board of Medical Examiners^ Time Remaining:
® Mark Comprehensive Basic Science Self-Assessment 1 hr 12 min 29 sec

49. A 70- year- old woman is brought to the emergency department 30 minutes after she was found unresponsive at home . She appears stuporous . Physical examination shows 2-mm-diameter pupils that are
reactive to light. Eye movements are full with ice water caloric stimulation. She withdraws the extremities symmetrically to painful stimuli. An MRI of the brain is shown ; the arrowheads indicate abnormalities.
Which of the foilowing is most likely to be present in this patient 2 months later ?

A ) Difficulty recognizing familiar faces


8 } Hesitant telegraphic language output
,

C ) Impaired abstraction and problem solving


O 0 ) Inability to learn and recall new facts
E ) Inappropriate , disinhiibited behavior

O 9 0 0
Previous *
Next Lab Values Calculator Review Help Pause
Exam Section 4: Item 50 ol 50 National Board of Medical Examiners^ Time Remaining:
® Mark Comprehensive Basic Science Self-Assessment 1 hr 12 min 26 sec

50 . A 34 -year- old woman comes to the physician because she has been feeling weepy and overwhelmed since delivering a healthy male newborn 6 weeks ago. She has had fatigue and irritability during this period , and she has had no interest in engaging in
activities she used to enjoy prior to the birth of the baby. She also has had difficulty sleeping because she is always listening for sounds indicating that her baby is awake . She says , ''My husband doesn't help me at all with the baby. We argue all the time
now. " She then says: " I feel guilty because I’m not enjoying my baby more . I don't know if I will be a good mother or not . ' Physical examination shows no abnormalities. Which of the following statements by the physician is most appropriate ?
1

A ) "Has it come as a surprise to you how hard parenting is ? Many people feel that way "
3 B ) " Tm concerned about how bad you've been feeling lately. Have you had any thoughts about death or wanting to be dead?"
o C ) "lrm sure that you'll feel better soon After alE look at your beautiful baby
. : .'
1

3 0 ) "Most new mothers feel this way, and sometimes it helps to see a therapist . Would you like me to give you a referral?"
E } "Your relationship with your husband sounds strained . \ don't blame you for being angry with him. "

O
Previous Next Lab Values Calculator
m
Review
O
Help
m
Pause

You might also like